Old Q&A – Female Reproductive System

A 34-year-old woman comes to the clinic because of left lower quadrant pain for the last 4 months. The pain is intermittent and seems to worsen during her periods. She has no significant past medical history and does not take any medications. She is sexually active with 1 partner and they use condoms for birth control. She has never been pregnant and has regular menstrual periods. Her last menstrual period was 17 days ago. Physical examination and vital signs are normal. Pelvic examination reveals mild tenderness in the mid and left side of the pelvis. There is no vaginal discharge or bleeding and the cervical os is closed. The most appropriate next step is to

  A. order a CT scan of the pelvis
  B. order ultrasonography of the pelvis
  C. prescribe nonsteroidal antiinflammatory medications for menstrual cramps
  D. prescribe oral contraceptives
  E. schedule a diagnostic laparoscopy
Explanation:

The correct answer is B. The most likely cause of this patient’s left sided pelvic pain is ovarian cysts. A follicular cyst occurs when the normal follicle does not shrink after the release of the egg. Follicular cysts are soft, have thin membrane walls, and contain clear fluid. They can rupture, causing sudden, severe pain that gradually goes away over several days. Usually, however, they spontaneously disappear over the course of 1 or more menstrual cycles. They can cause changes in periods and pelvic pain. Depending on the size of the cyst, it can just be observed for spontaneous resolution or treated with oral contraceptive pills. The best way to confirm the diagnosis of ovarian cysts is by ultrasonography of the pelvis.

A CT of the pelvis (choice A) is not the best way to image the ovaries or to confirm the diagnosis of ovarian cysts. A CT does not provide the resolution that ultrasonography does to evaluate the cyst in fine detail in order to ensure that it is a simple cyst as opposed to a more worrisome complex cyst.

Prescribing nonsteroidal antiinflammatory medications for menstrual cramps (choice C) is not appropriate in the management of this patient. She has had symptoms of intermittent pelvic pain for the last 4 months which makes menstrual cramps as the etiology of pain, less likely. It is important to further evaluate the pelvis for a cause of her symptoms and confirm the diagnosis of ovarian cysts. It is also important to rule out neoplastic causes of pain such as a cystadenoma or cystadenocarcinoma.

Prescribing oral contraceptive pills (choice D) is a reasonable option once the diagnosis of an ovarian cyst is made. However, it is important to make the diagnosis first with ultrasonography of the pelvis. If the cyst appears simple on ultrasonography and is 4 cm or less in size, it can be followed with serial ultrasonography for resolution. For simple uncomplicated ovarian cysts in the premenopausal period, treatment with the administration of high-dose monophasic oral contraceptive pills (>35 mcg estradiol) is recommended after consultation with a gynecologist to normalize cycles and to facilitate reduction in cyst size.

Diagnostic laparoscopy (choice E) allows direct visualization of the intraperitoneal contents including the ovaries which can add diagnostic yield. Surgical intervention and/or excision of relevant pathology may be accomplished by this method. This procedure is not the next appropriate step in the management of this patient. Ultrasonography of the pelvis usually provides sufficient information to make the diagnosis of simple uncomplicated ovarian cysts, without the need for direct visualization of the pelvic organs.

A 26-year-old woman, gravida 1, with insulin-dependent diabetes mellitus comes to the office at 33-weeks gestation, reporting decreased fetal movement for 1 day. She is found to have poor glucose control, with serial blood sugar levels greater than 150 mg/dL. The fetal heart rate on the nipple stimulation test is non-reactive. The baseline rate is 140/min, and late decelerations are observed in the first 30 minutes of the test with each contraction. The next step in management is

  A. a biophysical profile (BPP)
  B. delivery
  C. a nonstress test (NST)
  D. a contraction stress test (CST)
  E. a repeat nipple stimulation in 24 hours
Explanation:

The correct answer is A. Because there is a high false positive rate with CSTs (up to 50% falsely abnormal), the next step in the algorithm of fetal distress is to perform a BPP. This test consists of an evaluation of the fetal heart rate without any stressors (i.e., without an oxytocic agent) along with four other parameters derived from an obstetrical ultrasound within a given period of time. The four parameters are gross body movement, fetal tone, fetal breathing movement, and amniotic fluid index. Each parameter is given 2 points for a maximum of 10 points. A score of 6 or below, necessitates immediate delivery.

Delivery (choice B) in not indicated at this time, because a positive CST has a high false positive rate. Therefore, it would be premature to deliver an infant that is doing well at such an early gestational age.

NST (choice C) is incorrect because an NST gives you even less information about the status of the fetus than a CST. The NST is just an observation of the fetal heart rate as a function of time given without any stressors.

Contraction stress test (CST) (choice D) is incorrect. A CST can be in response to endogenous oxytocic agent (nipple stimulation test) or to an exogenous source such at oxytocin (Pitocin). The intention of CST is to test fetoplacental reserve by causing contractions of the uterus and thus decreasing blood and oxygen supply to the fetus. A negative CST is defined as no periodic decelerations in the fetal heart rate with satisfactory recordings and at least three adequate contractions. A positive CST is defined as persistent and consistent late decelerations of fetal heart rate noted with more than half of adequate contractions. A negative CST is reassuring and predicts fetal survival for up to 5-7 days after the test. A positive CST, however, can be falsely positive up to 50% of time.

Repeating the nipple stimulation in 24 hrs (choice E) is incorrect. Waiting a day, may be too late and repeating the nipple stimulation does not give you any extra information about the fetal well-being.

 

 

You are the doctor on call in the well baby nursery at the community hospital. One of the nurses calls you to ask about one of your patients. The baby is now 30 hours old and was born full term via vaginal delivery to a healthy 28-year-old mother. There were no complications at the delivery and the baby has been feeding well. The nurse is concerned that the baby looks “yellow”. You ask her to send for a bilirubin level. A few hours later she calls to tell you that the total bilirubin level has come back at 18 mg/dL with a direct bilirubin level of 0.6 mg/dL. The parents are now concerned about the baby’s discoloration. The most appropriate next step is to

  A. reassure the mother that this is completely normal and no additional studies or treatment are indicated
  B. repeat the bilirubin level immediately as the result must be a lab error
  C. start phototherapy and repeat the bilirubin level in 6 hours
  D. transfer to the nearest neonatal intensive care unit for an exchange transfusion
  E. wait 6 hours and repeat the bilirubin level
Explanation:

The correct answer is C. Jaundice is a common occurrence in the first week of life. “Physiologic jaundice” is usually transient and is due to an increased bilirubin load from increased red blood cell volume, decreased survival time of the red blood cell, and an increased enterohepatic circulation. Other factors include defective hepatic uptake of bilirubin and inadequate conjugation of bilirubin to bilirubin glucuronide for excretion. A normal cord blood bilirubin level is about 1.5 mg/dl, and a neonate’s serum bilirubin normally increases by a maximum of 5mg/dl in 24 hours to a maximum of about 15mg/dl at 3 days of life. A level of 18mg/dl at 30 hours of life is not normal and requires phototherapy to bring the level down. Phototherapy should bring the level down by 1-2mg/dl within 4-6 hours. The level should be checked at that time to ensure that the level has not continued to rise. Besides phototherapy, an investigation into the cause for the rapid rise of bilirubin should be undertaken. The most important consideration is the blood type of the mother and the blood type of the infant. A direct Coombs test should also be performed. Other things to keep in mind include a red cell defect such as an hemoglobinopathy, polycythemia, extravascular blood loss, bacterial sepsis, increased enterohepatic circulation (from an intestinal obstruction), disorder of bilirubin metabolism, or an endocrine disorder such as hypothyroidism.

Reassurance that this is completely normal (choice A) is incorrect. This baby needs to be treated for hyperbilirubinemia and more blood will need to be drawn to follow the levels. The mother should be reassured that this is a common occurrence, that is usually transient, and has no future implications on the baby’s development, if treated appropriately.

Repeating the labs (choice B) is not correct because although lab error is something to keep in mind when interpreting all test results, this baby is jaundiced and therefore it is unlikely to have a normal bilirubin level. Jaundice usually becomes apparent at levels of 5mg/dl in the neonate. As levels get higher, the jaundice usually progresses from the face to the trunk and then down the body.

An exchange transfusion (choice D) is not yet warranted in this baby. If the level were greater than 25mg/dl or started at greater than 20 mg/dl and did not improve with phototherapy, this baby would require an exchange transfusion. During this procedure, blood is removed from the baby (usually from an umbilical artery catheter), and the baby is transfused with cross matched blood through a venous catheter. The exchange is done in 15ml increments. There are multiple risks to the procedure including thromboemboli, dysrhythmias, hyperkalemia, hypernatremia, DIC, and transfusion reaction. And thus answer D is not correct.

Wait 6 hours and repeat the level (choice E) is incorrect because the bilirubin level is too high to allow you to wait and watch. If the bilirubin level is allowed to rise unchecked the baby is put at risk for kernicterus. When the level of unconjugated bilirubin reaches higher than 20mg/dl in a full-term neonate, it can become neurotoxic by accumulating in the basal ganglia, pons, or cerebellum causing kernicterus. Clinically, it manifests as a variety of neurologic symptoms ranging from lethargy and hypotonia to severe encephalopathy and death. At the present rate of rise, the bilirubin is likely to be higher than 20 mg/dlin 6 hours.

 

 

A 31-year-old woman comes to the emergency department because of heavy vaginal bleeding. She is 10-weeks pregnant, by her last menstrual period and has been getting routine prenatal care. She has been in good health and her pregnancy has been uneventful, until the heavy bleeding started the night before. She reports having to change a pad every hour and thinks she may even have passed blood clots. She has no significant past medical history and her only medication is a daily prenatal vitamin. She denies alcohol, cigarette, or drug use. Vitals are normal. Pelvic examination reveals an open cervical os and blood in the vaginal vault. Ultrasonography of the pelvis demonstrates a single endometrial canal containing echogenic material. The uterus is homogeneous and there is no evidence for an intrauterine pregnancy. The factor most likely contributing to this patient’s spontaneous abortion is

  A. a bicornuate uterus
  B. a chromosome abnormality of the fetus
  C. hypothyroidism
  D. an incompetent cervix
  E. a submucosal leiomyoma
Explanation:

The correct answer is B. Approximately 50% of early spontaneous abortions are due to chromosomal abnormalities of the fetus. Trisomy abnormalities account for 40-50% of these. The incidence of spontaneous abortions is estimated at 50% of all pregnancies.

Bicornuate uterus (choice A) is an anatomic abnormality of the reproductive tract due to the lack of fusion of the superior portion of the uterine body. This anomaly leads to difficulty in conception as well as an increased risk of a spontaneous abortion. This is not the likely cause of this patient’s spontaneous abortion because the ultrasound did not show two separate endometrial canals.

Hypothyroidism (choice C) and other endocrine abnormalities increase the risk of spontaneous abortion. This is not the likely cause in this case since the patient has no prior medical problems. It would be exceedingly unusual to have hypothyroidism suddenly present during pregnancy.

An incompetent cervix (choice D) is not the likely cause of a spontaneous abortion in this patient. The classic presentation of an incompetent cervix is a sudden expulsion of a normal sac and fetus between the 18th and 32nd week of pregnancy without antecedent pain or bleeding.

A submucosal leiomyoma (choice E) is a cause of difficulty in conception as well as spontaneous abortion. This is not a likely cause of abortion in this case because the ultrasound did not demonstrate any focal masses in the uterus.

 

 

A 33-year-old woman comes to the office for a periodic health maintenance examination. She has no specific complaints. Her last menstrual period began 10 days earlier. Physical examination is unremarkable. Pelvic examination reveals a mobile mass in the left adnexa and an ultrasound shows that is it is a 4 cm unilocular, homogeneous, fluid-filled mass. The most appropriate next step is to

  A. aspirate the mass under ultrasound guidance
  B. order serum α-fetoprotein and human chorionic gonadotropin levels
  C. order serum CA-125 concentration
  D. prescribe oral contraceptive pills
  E. repeat the examination in 2-3 months
  F. schedule a laparoscopy
Explanation:

The correct answer is E. In premenopausal women, simple cystic masses are usually benign, functional ovarian cysts that resolve spontaneously. Functional ovarian cysts are thought to be formed when an ovarian follicle fails to rupture during maturation. Reassurance and reexamination in 2-3 months are recommended.

Since this cyst is most likely a functional ovarian cyst that will probably resolve spontaneously, it is inappropriate to aspirate the mass under ultrasound guidance (choice A). If the mass happens to be malignant and you aspirate it, this can lead to leakage, peritonitis and seeding of the peritoneal cavity with the tumor.

Germ cell ovarian tumors are associated with elevated levels of α-fetoprotein and human chorionic gonadotropin (choice B). This patient most likely has a functional ovarian cyst rather than a rare, germ cell tumor.

It is unnecessary to order serum CA-125 (choice C) because this adnexal mass is most likely a functional ovarian cyst, as opposed to an ovarian malignancy. If the cyst does not resolve, this test may be indicated in the future.

Oral contraceptive pills (choice D) are often prescribed for a functional ovarian cyst that does not resolve spontaneously. It may be appropriate after reevaluating this patient in a few months if the cyst is still present.

At this time, a laparoscopy (choice F) for an asymptomatic simple cyst in a premenopausal woman is too aggressive since this is most likely benign. This may be the last resort if the cyst does not resolve spontaneously after a trial of oral contraceptive pills.

 

 

A 28-year-old gravida 3, para 2 woman comes to the clinic for prenatal care at 11-weeks gestation. Her medical and surgical history are unremarkable, although she relates a social history significant for alcohol consumption. She drinks 1-2 glasses of wine with lunch and 3-4 glasses of wine with and after dinner on most nights. Given her history, her fetus is at greatest risk for

  A. a bowel obstruction
  B. a cardiac defect
  C. cleft lip and palate
  D. macrosomia
  E. tall stature
Explanation:

The correct answer is B. Any fetus exposed to alcohol, especially during the first trimester, is at risk for fetal alcohol syndrome (FAS). Cardiac malformations, especially ventricular septal defect (VSD), are noted with increased frequency among infants exposed to excessive levels of alcohol in utero. Additional characteristics of children with FAS may include CNS impairments such as microcephaly, mental retardation, and attention deficit disorders. For women with “moderate” alcohol intake, defined as 1-2 oz of absolute alcohol per day (i.e., 2-4 drinks per day), the risk of FAS is 10%. This woman consumes 4-6 drinks per day and thus places her fetus at risk for some component of FAS.

Bowel obstruction (choice A) is incorrect because it is not found in FAS. Bowel obstruction, which can occur with duodenal atresia, is found with increased frequency in a fetus with Down’s syndrome, not FAS.

Cleft lip and palate (choice C) is incorrect. Facial dysmorphia found in FAS includes low-set ears, smooth philtrum, and short palpebral fissures.

Macrosomia (choice D) is incorrect. Growth restriction (not macrosomia or tall stature) is found either prenatally or postnatally in FAS.

Tall stature (choice E) is also incorrect. Children with FAS are small for gestational age and growth restricted.

 

 

A 26-year-old African American woman walks into the emergency department where you are working. She is 37-weeks pregnant and is complaining of a severe headache for the past 24 hours. She also tells you that she has noticed that the vision in her right eye has been extremely blurry since she woke up this morning. She states that her pregnancy has been uneventful and that she receives prenatal care at the hospital clinic. The card she is carrying with her indicates that all prenatal tests were within normal limits. Three consecutive blood pressure readings 15 minutes apart are 156/102 mm Hg, 164/112 mm Hg, and 144/98 mm Hg. The nurse informs you that her bedside urine dipstick reveals 3+ proteinuria. On physical examination you find a mild systolic ejection murmur and 2+ pitting edema of her lower extremities. A sterile vaginal exam reveals a long and closed cervix. Tocodynomometer shows irregular uterine contractions every 8-10 minutes. The external fetal heart tracing is reassuring. The most appropriate next step in this patient’s management is

  A. an emergent ophthalmology consult
  B. intravenous administration of magnesium sulfate and induction of labor
  C. intravenous administration of magnesium sulfate and a shot of intramuscular terbutaline to quiet her uterine contractions
  D. prolonged external fetal heart monitoring
  E. ultrasound examination of the fetus to evaluate amniotic fluid index and fetal well being
Explanation:

The correct answer is B. This patient has preeclampsia; (hypertension that results as a consequence of pregnancy and regresses postpartum, proteinuria, and/or pathologic edema (i.e., facial/hand)). Preeclampsia may progress to eclampsia, which is a generalized seizure. Patients at term (37 weeks and on) who present with preeclampsia should be induced and given magnesium sulfate at the time of induction as a seizure prophylaxis.

In some cases patients who are preeclamptic experience visual changes secondary to retinal detachment. This patient is complaining of blurred vision, which may indicate that she needs an ophthalmologic evaluation (choice A). However, the top priority is delivery.

This patient is having irregular contractions. These are normal for any term patient and even if you were not going to induce her labor, you would not want to give her a tocolytic such as terbutaline (choice C).

Fetal evaluation is important in preeclamptic patients (choice D). In addition, many preeclamptic patients present with a decreased amniotic fluid index (oligohydramnios) secondary to placental insufficiency (choice E). However, delivery is again the top priority.

 

 

A 2-day-old female infant in the neonatal unit has a distended abdomen and has not passed meconium since birth. The child was delivered vaginally at term, but her birth was induced with magnesium sulfate, because the mother was diagnosed with preeclampsia. The infant’s vital signs are: temperature 38.1 C (100.6 F), blood pressure 70/40 mm Hg, pulse 130/min, and respirations 22/min. Physical examination is significant for a distended abdomen. An abdominal x-ray demonstrates a “bubbly” bowel gas pattern on the left side of the abdomen. The next step in the management of this patient is

  A. bowel rest
  B. contrast enema
  C. intravenous fluids
  D. laparotomy
  E. an ultrasonography
Explanation:

The correct answer is B. Clinically, the paucity of meconium, and the presence of abdominal distension in a neonate, are suspicious for meconium plug syndrome. The “bubbly” appearance on the left abdomen reinforces this consideration. Meconium plug syndrome is more common in infants after labor induction with magnesium sulfate. A water-soluble contrast enema will reveal a spindly plug of meconium in the colon, and often will serve also as a therapeutic enema. Differential considerations include small left colon syndrome, which may coexist with meconium plug syndrome. Also, if therapeutic contrast enema fails to resolve symptoms, Hirschprung’s disease or cystic fibrosis must be considered. Do not confuse these entities with meconium ileus.

Conservative treatment such as bowel rest (choice A) is not the appropriate management. A contrast study is necessary to evaluate for meconium plug syndrome. Failure to diagnose and treat may result in perforation.

Conservative treatment such as intravenous fluid resuscitation (choice C) is not appropriate. A contrast study is necessary to evaluate for meconium plug syndrome.

A laparotomy (choice D) is not indicated for meconium plug syndrome or small left colon syndrome.

An ultrasonography (choice E) is a useful screening tool for intussusception, but not for meconium plug syndrome.

 

 

A 22-year-old college student comes to the emergency department with a severe right lower quadrant pain. She says that the pain started approximately 6 hours ago and has progressively worsened. She has no significant medical problems and her only medication is oral contraceptive pills. She is sexually active with 1 partner, her boyfriend. Her last menstrual period was 2 weeks ago. Vital signs are: temperature 37.0 C (98.6 F), blood pressure 120/70 mm Hg, and pulse is 80/min. Abdominal examination is significant for focal tenderness in the right lower quadrant. Pelvic examination reveals exquisite tenderness in the right adnexa, a closed cervical os, and clear vaginal discharge. Laboratory studies show:

The most likely etiology of this patient’s symptoms is

  A. acute appendicitis
  B. diverticulitis
  C. ovarian torsion
  D. a ruptured ectopic pregnancy
  E. a tuboovarian abscess
Explanation:

The correct answer is C. Ovarian torsion is the most likely cause of abdominal pain localized to the right adnexa in this patient. She has no significant medical problems and presents with an acute onset of right lower quadrant pain in the absence of fever or other laboratory signs of an infectious process. Ovarian torsion often occurs in the setting of other ovarian pathology such as a cyst or other lesions in the ovary that causes it to twist upon its vascular supply. Ovarian torsion is quickly diagnosed by ultrasonography of the pelvis with the demonstration of absence of flow to the ovary. This is an urgent diagnosis and surgery is required to restore blood flow.

Acute appendicitis (choice A) is not a likely cause of abdominal pain in this patient. Acute appendicitis is an infectious process that results from obstruction of the appendix (usually by a fecalith) and subsequent bacterial infection. It usually presents with right lower quadrant pain, fever, and leukocytosis. The presence of peritoneal signs such as rebound tenderness raises the suspicion for a ruptured appendix.

Diverticulitis (choice B) is not a likely cause of abdominal pain in this patient. Diverticulitis is an infectious process that typically affects older patients who have diverticulosis. This most often affects the sigmoid colon, although diverticulosis can occur anywhere in the colon. Signs and symptoms of diverticulitis include lower abdominal pain, fever, nausea, vomiting, and leukocytosis.

A ruptured ectopic pregnancy (choice D) is not a likely cause of abdominal pain in this patient. This diagnosis must be considered in any pregnant woman who presents with abdominal pain, vaginal bleeding, and symptoms of shock. It is a surgical emergency. This patient has a negative serum Beta-HCG, which rules out the possibility of an intrauterine or ectopic pregnancy.

A tuboovarian abscess (choice E) is not a likely cause of abdominal pain in this patient. A tuboovarian abscess is an advanced form of pelvic inflammatory disease most often caused by the spread of bacteria from the lower genital tract. The most common bacterial pathogens are anaerobic. Risk factors for pelvic inflammatory disease include those associated with an increased risk of contracting a sexually transmitted disease, early age of first sexual encounter, multiple sexual partners, history of sexually transmitted disease, and douching. In addition, women using IUDs are at increased risk for pelvic inflammatory disease and tuboovarian abscess. Symptoms of tuboovarian abscess include pelvic pain, fever, leukocytosis, and abnormal cervical or vaginal discharge in the setting of pelvic inflammatory disease.

 

 

A 46-year-old woman comes to the office for a periodic health maintenance examination. She is married and has worked in the local library for 15 years. She states that she has no medical problems, runs 3 miles every other day, and feels generally well. She takes a multivitamin, as well as 1000 mg of calcium every day. She has never smoked cigarettes, and reports drinking wine with dinner when she goes to a restaurant. Her only surgery was a bilateral tubal ligation 10 years ago after her third child. When questioned about her menstrual periods, she said that she has experienced a 28-30 day cycle for years, with three days of bleeding. She does mention that for the past three months her periods have been different, with bleeding about every 15 days, lasting 2 days. A physical examination and a pelvic examination reveal no abnormal findings. The most important next step in this woman’s care is to

  A. cycle her on a low-dose combination oral contraceptive pills and give her some extra iron
  B. determine FSH and LH levels
  C. send her for an ultrasound evaluation of her pelvic anatomy
  D. perform an office endometrial Pipelle biopsy
  E. perform a routine Pap smear and send her for a mammogram
Explanation:

The correct answer is D. With an abnormal bleeding pattern in a woman over 35, the most important thing to rule out is endometrial hyperplasia or cancer. The first step in a workup for endometrial hyperplasia is to perform an easy office Pipelle biopsy of the endometrium and send it for pathologic evaluation. If this returns as normal, you can consider other reasons for her bleeding, which may include submucosal fibroids, polyps, anovulation, or perimenopause.

Once you rule out other reasons for her bleeding, such as endometrial hyperplasia, fibroids, or a polyp, you can cycle her on OCPs to regulate her bleeding (choice A). Many perimenopausal women are cycled on low-dose contraceptive pills as well.

An altered FSH/LH (choice B) may indicate that she is perimenopausal; however, abnormal bleeding in a patient her age should be evaluated with a biopsy.

An ultrasound evaluation (choice C) would be reasonable to evaluate her uterus for any fibroids, and a saline infused sonogram would show polyps.

Clearly this woman also needs a Pap smear (choice E), but as a physician the most important thing to do is rule out cancer with a biopsy. A mammogram is not routinely recommended by the United States Preventive Services Task Force until age 50.

 

 

A 16-year-old girl comes to the clinic because of a 2-week history of nausea and vomiting in the morning before school. The nausea comes on as soon as she wakes up, and is generally relieved after she “throws her guts up.” She attends school and is able to function at her evening job as a pharmacy clerk. She denies any weight loss. She is sexually active with “many,” different partners and does not use any form of contraception. She also admits to injection drug use. She is unsure of the exact date of her last menstrual period, but thinks that it was about 3-4 weeks ago. Her chart indicates that she has received the following vaccinations: IPV, MMR, and DaPT. Her blood pressure is 120/80 mm Hg and pulse is 65/min. Physical examination is normal. A pregnancy test is positive. You tell her that she is pregnant and she tells you that she is going to keep the baby and has no plans to decrease her sexual activity or drug use. She agrees to undergo HIV testing in addition to routine tests. Laboratory studies show:

At this time you should:

  A. admit her to the hospital for the treatment of hyperemesis gravidarum
  B. advise her to use oral contraception pills to prevent future pregnancies
  C. have her write a statement about her plans to continue unsafe sexual activity and drug use and place it in her chart
  D. recommend the hepatitis B vaccine
  E. tell her that she must discontinue her drug use if you are going to continue to treat her
Explanation:

The correct answer is D. This pregnant patient engages in behavior that puts her at a high risk of contracting hepatitis B, (and other diseases such as HIV, hepatitis C, and STDs). Her vaccination history does not include the hepatitis B vaccine, and her laboratory studies indicate that she does not have antibodies to the disease. She should be given the vaccine at this time, even though she is pregnant, because there have been virtually no adverse effects on the unborn fetus of mothers who have received the vaccine during pregnancy. Pregnancy is not a contraindication to immunization.

Hyperemesis gravidarum (choice A) is a severe form of nausea and vomiting during pregnancy that affects about 4 in 1,000 women. Individuals with hyperemesis gravidarum have metabolic abnormalities, dehydration, and weight loss. Treatment includes intravenous crystalloid fluids, with necessary electrolytes. This patient has typical “morning sickness,” which is nausea and vomiting that affects the majority of pregnant women.

After she delivers this baby, it may be appropriate to advise her to use oral contraception pills to prevent future pregnancies (choice B). However, at this time, she is pregnant and should not use OCPs. She should, however, use condoms to reduce the risk of acquiring an infectious disease.

It is important to document in the patient’s chart that she was advised to stop all high-risk behavior (so the physician will limit liability if she says that she was not warned of the affects on the fetus). However, it is not necessary to have her write a statement about her plans to continue unsafe sexual activity and drug use, and place it in her chart (choice C).

Since you want to establish a trustworthy physician-patient relationship, it would be unadvisable to tell her that she must discontinue her drug use if you are going to continue to treat her (choice E). You should try to explain to her why her behavior might be harmful to her and her unborn child, but ultimately it is your duty to treat her, even if you do not approve of her decisions.

 

 

A 39-year-old woman comes to the office because of right lower quadrant and thoracolumbar back pain. She has no significant past medical history and does not take any medications. She denies cigarette or alcohol use. A CT scan of the abdomen and pelvis is obtained and the study is shown.

The preventive measure most likely to have prevented this outcome is a

  A. chest auscultation
  B. clinical breast exam
  C. clinical thyroid exam
  D. mammography
  E. self breast exam
Explanation:

The correct answer is E. The history of thoracolumbar back pain and the CT findings of a sclerotic metastasis to the right hemivertebra are diagnostic for metastatic disease. The most common cancers for women in this age group are lung and breast cancer. There is no effective screening tool for lung cancer. For breast cancer, a self breast exam picks up most breast cancers before presentation. It is estimated that more than 80% of cancerous lumps are found by women themselves. While the recommendations are controversial with many groups having different guidelines, The American Cancer Society (ACS) guidelines for the early detection of breast cancer in asymptomatic women are: (1) Women 30 years of age and older should perform breast self examination every month. (2) Women, 20-39, should have a physical examination of the breast every 3 years performed by a health care professional such as a physician, physician assistant, nurse, or nurse practitioner. (3) Women 40 and older should have a physical examination of the breast every year performed by a health care professional such as a physician, physician assistant, nurse, or nurse practitioner. (4) Women 40 years of age and older should have a mammogram every year.

A chest auscultation (choice A) rarely detects lung cancer. Screening chest x-rays have not been shown to be an effective screening tool.

A clinical breast exam (choice B) is an important part of screening for breast cancer, but the self breast exam picks up most cancers.

A clinical thyroid exam (choice C) is important, but thyroid cancers are rare compared to breast cancer.

A mammography (choice D) is an important part of screening for breast cancer, but the self breast exam picks up most cancers. Also, screening mammography is generally not recommended until age 40.

 

 

A 2-day-old male newborn in the neonatal unit has abdominal distension. The birth was a 39-week normal vaginal delivery. There is a family history of cystic fibrosis. Vital signs are: temperature 38.1 C (100.6 F), blood pressure 74/40 mm Hg, pulse 132/min, and respirations 21/min. On physical examination, the abdomen is distended and bowel sounds are high pitched. An x-ray of the abdomen shows multiple dilated loops of both small and large bowel. The next step in management of this newborn is to

  A. administer oral stool softener
  B. order a barium enema
  C. order a CT scan of the abdomen
  D. perform a sweat test
  E. send CFTR gene analysis
Explanation:

The correct answer is B. This child has clinical and radiographic evidence of meconium ileus. This occurs when hard meconium impacts in the ileum. Cystic fibrosis (CF) is a risk factor for meconium ileus. Indeed, this entity occurs in adults with CF as well. Barium enema reveals a microcolon and filling defects in the ileum consistent with meconium plugs. The enema is therapeutic as well. The differential consideration is ileal atresia; the 2 entities are distinguished by a barium enema.

Oral stool softeners (choice A) may be used in cases of meconium illness, but mechanical disimpaction with a contrast enema is necessary.

A CT scan of the abdomen (choice C) is a second-line study used for problem solving. The first-line modalities in the newborn with suspected meconium ileus are abdominal x-rays and contrast enemas.

The sweat test (choice D) along with nasal potential testing are 2 initial screening tools to detect cystic fibrosis. These must be confirmed with genotyping. This child needs immediate workup for his gastrointestinal problems and that must take priority over the sweat test.

CFTR gene analysis (choice E) is the definitive test to evaluate for cystic fibrosis. This child needs immediate workup for his gastrointestinal problems and that must take priority over gene analysis.

 

 

A 17-year-old runaway comes to the emergency department because of a 24-hour history of lower abdominal pain and vomiting. She tells you that she hates doctors and hospitals and is only here because another girl on the street told her that “this may be serious.” She asks you to give her medicine quickly so she can leave. She is sexually active with multiple partners and she “occasionally” uses condoms for contraception. She lives “on the streets” and begs for money at the doorways of banks. She has not received any medical care in 6 years. Her last menstrual period was 9 days ago. She is unsure if she ever had a sexually transmitted disease in the past. Her temperature is 38.8 C (101.8 F), blood pressure is 110/70 mm Hg, and pulse is 65/min. Physical examination shows bilateral lower abdominal tenderness, but rebound tenderness and guarding are absent. Pelvic examination shows cervical motion tenderness, adnexal tenderness, and a yellowish-white cervical discharge. There are no palpable masses. A urine pregnancy test is negative. Cervical cultures are taken and sent to pathology for evaluation. The erythrocyte sedimentation rate and C-reactive protein level are elevated. The most appropriate next step is to

  A. admit her to the hospital and begin cefoxitin and doxycycline therapy, intravenously
  B. admit her to the hospital and prepare her for an immediate operation
  C. do a culdocentesis
  D. prescribe ofloxacin and metronidazole therapy, orally, and discharge her
  E. try to contact her male sexual partners for evaluation and treatment
Explanation:

The correct answer is A. The diagnosis of pelvic inflammatory disease (PID) is typically based on clinical findings such as lower abdominal tenderness, cervical motion tenderness, adnexal tenderness, a vaginal or cervical discharge, fever >38.3 C, (101.0 F), elevated erythrocyte sedimentation rate and C-reactive protein, and laboratory documentation of N. gonorrhoeae or C. trachomatis. Treatment should be given even before the culture results return to reduce the risks of infertility and progressive infection. Admission to the hospital is important for this patient because compliance and follow-up cannot be assured since she is homeless and has already said that she hates doctors and hospitals. There is a chance that this patient may take the prescription and not return, so admission is indicated.

It is not necessary to admit her to the hospital and prepare her for an immediate operation (choice B). She most likely requires intravenous antibiotic therapy, not surgery. Surgery may be indicated if there is a suspicion of appendicitis, a ruptured ovarian cyst, or an ectopic pregnancy. However, this patient seems to have a classic case of PID. A tuboovarian abscess (TOA) may also require surgical evaluation and drainage. Patients with a TOA often have fever >39.5 C, (103.1F), tachycardia, and severe pelvic pain.

A culdocentesis (choice C) is not necessary at this time. This procedure, which is transvaginal sampling of the cul-de-sac, may be performed to support the diagnosis of an ectopic pregnancy if clotted blood is found. It may also be used to obtain bacteriologic samples. It is not indicated at this time because there is a cervical discharge present that has been sent to pathology for evaluation.

If this patient with PID was known to be a compliant patient and follow-up within 72 hours can be assured, it is appropriate to prescribe ofloxacin and metronidazole therapy, orally, and discharge her (choice D). Since she is homeless and already said that she hates doctors and hospitals, compliance and follow-up cannot be assured, and so she should be treated with in-patient therapy. Admission for all adolescents with PID used to be recommended, but this is no longer true because there is not a lot of data to support this recommendation. However, admission is recommended for the following individuals including pregnant patients with PID, an abscess or peritonitis, HIV patients with PID, and all those that fail outpatient therapy.

Evaluating and treating her male sexual partners (choice E) is important because this condition is due to a sexually transmitted disease, however, this patient requires immediate therapy and so contacting sexual partners should wait until therapy for this patient is initiated.

 

 

You are called to the labor floor in a community hospital because a 25-year-old patient of yours, who is 37 weeks pregnant, just presented to the triage area complaining of painful uterine contractions every 3 minutes lasting 90 seconds. Fetal well-being is assured via external fetal heart monitoring, and a sterile vaginal exam reveals a cervix that is 6-cm dilated. Her blood pressure is 90/50 mm Hg and urine dip is negative. You check her prenatal chart. She has had an uneventful normal pregnancy course, with prenatal care starting at 8 weeks. Prenatal labs were significant only for rubella non-immune and group B Streptococcus in a urine culture at 28 weeks. In addition to general intravenous hydration, the most appropriate management at this time is to administer

  A. ampicillin, intravenously
  B. magnesium sulfate, intravenously
  C. meperidine, intravenously
  D. oxytocin, intravenously
  E. Rubella vaccine
Explanation:

The correct answer is A. This is a full-term patient in a normal progressive labor pattern with a reassuring fetal heart tracing. She has risk factors for Group B Strep infection with a positive urine culture during pregnancy. The American College of Obstetrics and Gynecology recommends Group B Strep prophylaxis in labor for patients with risk factors. The treatment is penicillin or ampicillin in active labor.

Magnesium sulfate (choice B) is given to pre-eclamptic patients for seizure prophylaxis. This patient has normal blood pressure and no proteinuria, and is therefore not pre-eclamptic.

IV meperidine (choice C) can be used for pain relief. It is usually used in early labor (prior to 4-cm dilation), and is not as appropriate as choice B.

IV oxytocin (choice D) is often used for labor augmentation in laboring patients who are in a dysfunctional labor pattern. This patient is having regular contractions and has cervical change. Pitocin is not necessary at this point.

Rubella vaccine (choice E) is given post partum.

 

 

A 12-year-old pregnant girl comes to the clinic because of a 3-day history of a fever and an “itchy rash.” Her little brother had a similar rash 2 weeks earlier. She has received routine prenatal care and she has had an uneventful pregnancy so far. Her temperature is 38.3 C (101.0 F). Physical examination shows a generalized vesicular rash in various stages of evolution. There are vesicles and crusted lesions on her arms, legs, trunk, and face. A complete blood count and liver function tests are normal. At this time you should

  A. administer the varicella vaccine
  B. admit her to the hospital for intravenous acyclovir therapy
  C. advise her to stay home from school until all lesions are dried and crusted
  D. recommend salicylates for control of fever
  E. tell her that she is no longer contagious since the rash has already appeared
Explanation:

The correct answer is C. This patient most likely has chickenpox, which is caused by the varicella-zoster virus. It is recommended that all children with the chickenpox stay home from school until all lesions are dried and crusted. Fetal infection may occur and can result in limb atrophy, skin scarring, central nervous system complications, and even death. Some believe that oral acyclovir should be given to pregnant women with varicella, but this issue is controversial because the risks and benefits to the fetus are unknown. The varicella-zoster immune globulin should be given to pregnant women without a history of chickenpox provided that significant exposure to the virus has occurred. It is not routinely given to pregnant women who have the infection.

The varicella vaccine (choice A) is a live-attentuated preparation that is recommended for healthy individuals 12 months or greater who have not had varicella. It is not given to individuals with the disease.

Admitting her to the hospital for intravenous acyclovir therapy (choice B) is sometimes used for pregnant patients with serious complications of varicella. This patient seems to have an uncomplicated case of varicella (she does not seem to have complications such as thrombocytopenia, hepatitis, meningitis, encephalitis, or arthritis).

It is inappropriate to recommend salicylates for control of fever (choice D) because salicylates have been linked to Reye syndrome when given during a varicella illness.

If she had erythema infectiosum, it would be correct to tell her that she is no longer contagious since the rash has already appeared (choice E). However, this patient most likely has chickenpox and is therefore considered contagious until all the lesions are dried and crusted.

 

 

A 31-year-old woman and her husband have been unable to conceive for the past 3 years. The woman reports that for 10 years, she has had irregular menses, which occur between 21 to 60 days apart. She states that she is otherwise in good general health and that she and her husband have been competing in marathons for over 8 years. Her physical examination is normal. One year ago, her husband underwent semen analysis, which was normal. The study that would most likely be most informative about this patient’s infertility is

  A. basal body temperature record
  B. a CT of the sella turcica
  C. a hysterosalpingogram
  D. laparoscopy
  E. thyroid stimulating hormone (TSH) level
Explanation:

The correct answer is A. A chart recording a woman’s basal body temperature will indicate whether a potential ovulatory problem may exist. If a biphasic monthly temperature pattern is found, it confirms luteinization and ovulation. Female marathon runners are known to commonly have infrequent or absent menses and ovulations. Women who have very low body fat and/or participate in intense exercise are known to have a decrease in gonadotropin releasing hormone (GnRH), which in turn results in a diminished release of follicle stimulating hormone (FSH) and leuteinizing hormone (LH). Ovulatory dysfunction is responsible for approximately 20-25% of infertility cases.

A CT of the sella turcica (choice B) is performed as an initial test to evaluate for pituitary masses. It is not necessary at this time because the patient’s infertility is most likely due to anovulation related to her long distance running.

A hysterosalpingogram (choice C) involves an injection of dye into the uterine cavity in order to evaluate the contour of the uterine cavity, the patency of the fallopian tubes, and the ability of the dye to spill freely into the pelvis. It is useful in the diagnosis of structural causes of infertility such as congenital malformations of the uterus, submucosal leiomyomas, and tubal occlusions. It is not necessary at this time because the patient’s infertility is most likely due to anovulation related to her long distance running.

Laparoscopy (choice D) is not indicated at this time. This procedure complements information obtained in a hysterosalpingogram by revealing other tubal abnormalities, such as small filmy adhesions or peritubal cysts, that may not be observed on a hysterosalpingogram. Laparoscopy can also document endometriosis, which has a strong association with infertility.

Thyroid stimulating hormone (TSH) levels (choice E) are useful in the evaluation of amenorrhea. It is not necessary in this patient, since she is most likely anovulatory secondary to her long distance running.

 

 

A 45-year-old man with insulin dependent diabetes mellitus, peptic ulcer disease, hypercholesterolemia, and a motorcycle accident 10 weeks prior to this admission, has been on the telemetry floor for the last 3 days with shortness of breath and chest pain. His cardiac workup has been negative. An echocardiogram is pending. Vital signs have been stable with the exception of nightly low-grade fevers. There is no previous history of cardiopulmonary problems. During the accident he sustained multiple extremity fractures, and a pelvic fracture that required an external fixator for stabilization. The pin insertion sites for the external pelvis fixator became infected and the hardware was removed 10 days prior to this admission. He has 30 days of intravenous antibiotics remaining. On morning rounds, he complains of increased left wrist pain and swelling. He states that when he tries to hold his coffee cup the wrist pain increases. He denies any left hand or finger paresthesias. The left distal radius was fractured in the motorcycle accident. The left arm cast was removed 1 week ago after non-surgical management. He denies any new left wrist trauma. Physical examination reveals a left wrist with diffuse soft tissue swelling, that is fluctuant dorsally, with mild tenderness to palpation. Mild erythema and warmth are noted dorsally at the left wrist. Passive wrist flexion and extension exacerbates the wrist pain. The neurovascular examination is unremarkable in the left upper extremity. There is epitrochlear and axillary lymphadenopathy. The next best step in treating the left wrist pain, after x-rays are completed, is

  A. application of ice to the left wrist and elevation of the wrist. If the x-rays reveal any abnormalities then treat accordingly with a cast or splint and schedule orthopaedic follow up in 1 week
  B. aspiration of the fluctuant region of the left wrist; send the fluid for culture, sensitivity, and stat Gram stain; if the Gram stain is positive then plan for operative irrigation and debridement of the left wrist
  C. aspiration of the fluctuant region of the left wrist and the wrist joint. If the fluid has any color component to it, then plan for operative irrigation and debridement of the left wrist. Send the fluid for culture and sensitivity, a stat Gram stain, and ensure broad-spectrum antibiotic coverage
  D. if the x-rays are negative, then apply a resting splint to the left wrist; order occupational therapy to begin daily left wrist and hand therapy
  E. if the x-rays are negative, then aspirate the fluctuant region of the left wrist; send the fluid for a Gram stain, culture, and sensitivity; adjust the current intravenous antibiotic regimen pending the culture results
Explanation:

The correct answer is C. This patient clinically has a septic left wrist requiring surgical management. The aspiration confirms clinical suspicion and allows cultures to be sent preoperatively. Gram stains are not reliable. As an example, infected total joint arthroplasties have positive Gram stains less than 10% of the time. A turbid aspiration regardless of the Gram stain result would ensure an operative irrigation and debridement. The mechanism of the infection is hematogenous. Healing fractures, such as the left radius, have an increased blood flow. The infected pin sites at the pelvis resulted in transient bacteremia and easily seeded the wrist. The infection required weeks before it had any clinical signs. . Synovial joint fluid is clear in color and an aspiration with any color component suggests infection. There are exceptions to interpretations of aspirate color, but in this scenario, the infection is quite evident. Clinically, this patient has a septic joint and the aspiration confirms the diagnosis. Surgical debridement is the best management plan.

Apply ice to the left wrist and elevate. If the x-rays reveal any abnormalitiesy then treat accordingly with a cast or splint. Schedule an orthopaedic follow-up in 1 week (choice A) is incorrect as this is not a new fracture or sprain. The infection requires emergent surgical debridement.

Gram stains (choice B) are unreliable. Surgical decisions are based upon the history and physical examination.

A warm, tender, fluctuant joint in a patient with this history and physical exam, has a septic joint until proven otherwise. Splinting and hand therapy (choice D) are inadequate and not indicated. Aspiration confirms the diagnosis. Surgical debridement is the best management plan.

Aspiration and antibiotics (choice E) are not enough. A fluctuant region has little to no circulation and very little if any antibiotics will reach this region. Surgery (i.e., irrigation and debridement) is the only definitive treatment.

 

 

A 25-year-old comes to the emergency department because of severe right-sided lower abdominal pain for the past 12 hours. She has also experienced fever and chills, but no change in appetite. She is sexually active with 3 different partners and they use the “withdrawal” method for contraception. Her regular menstrual period began 4 days ago. Her temperature is 39 C (102.2 F), blood pressure is 120/80 mm Hg, pulse is 75/min, and respirations are 20/min. Physical examination shows right-sided lower abdominal tenderness with no rebound or guarding. Pelvic examination shows cervical motion tenderness and a purulent discharge. A Gram stain of the discharge shows Gram-negative diplococci within polymorphonuclear leukocytes. A urine pregnancy test is negative. You prescribe a 14-day course ofloxacin and metronidazole. She agrees to be compliant with this therapy and you send her home with a follow-up visit in 48 hours. She returns to the emergency department in 12 hours with one of her sexual partners because of worsening abdominal pain. Her temperature is 39.3 C (102.8 F), blood pressure is 100/60 mm Hg, pulse is 130/min, and respirations are 28/min. Abdominal examination shows severe tenderness with guarding and rebound. At this time you should

  A. add cefoxitin to her treatment regimen
  B. admit her to the hospital and prepare her for immediate surgery
  C. admit her to the hospital for observation
  D. evaluate and treat her sexual partner
  E. measure the serum β-subunit of human chorionic gonadotropin concentration
  F. order a CT scan of the pelvis
Explanation:

The correct answer is B. This patient with pelvic inflammatory disease (PID) most likely has a ruptured tuboovarian abscess (TOA), peritoneal inflammation and requires immediate surgery. The abscess needs to be removed before it leads to more severe peritonitis and death.

Adding cefoxitin to her treatment regimen (choice A) is incorrect. In a patient with PID and a TOA you need to make sure that she has effective anaerobic coverage, which she does because she was given metronidazole.

Admitting her to the hospital for observation (choice C) is incorrect because as stated above, she needs immediate surgery because of a TOA.

While it is extremely important to evaluate and treat her sexual partner (choice D), this patient has a ruptured TOA, which is a surgical emergency and requires immediate attention. Evaluating and treating the partner can wait until the proper arrangements are made for this patient.

It is unnecessary to measure the serum β-subunit of human chorionic gonadotropin concentration (choice E) because her regular menstrual period began a few days ago and a pregnancy test done 12 hours ago was negative.

This patient most likely has a ruptured TOA that requires immediate surgical attention and ordering a CT scan of the pelvis (choice F) will only waste important time.

 

 

A 59-year-old woman comes to the office for a periodic health maintenance examination. She and her 55-year-old sister, who is also a patient of yours, are concerned about their risk for breast cancer because they have been hearing so much about it on television. She says, “it seems like every woman over 40 has it today.” The patient’s menstrual period began at age 16 and she had a total abdominal hysterectomy and bilateral oophorectomy at age 43. She had one child at age 38. She eats a low fat diet with lots of fruits and vegetables and does not take hormone replacement therapy. Her sister’s menstrual period began at age 12 and menopause began at age 53. She had two children at ages 18 and 24. She eats lots of red meat and potato chips and has been taking hormone replacement therapy since menopause began. Comparing each detail individually, the factor in the patient’s history that increases her risk for breast cancer is

  A. age at first pregnancy
  B. age of menarche
  C. age of menopause
  D. diet
  E. lack of hormone replacement therapy (HRT)
Explanation:

The correct answer is A. Since breast cancer is a hormone-dependent disease, factors that are associated with increased hormonal exposure tend to increase the incidence, while factors that decrease exposure tend to decrease the incidence. Women who have their first full-term pregnancy before 18 years old, have a lower risk of breast cancer than those who have never been pregnant or those who wait until after they are 35 years old (choice A).

Women who begin menstruating at 12 years of age have a greater risk than women who began at age 16 (choice B).

Women who go into menopause naturally or surgically before 52 years old, (median age of menopause), have a lower risk of breast cancer (choice C).

Dietary factors (choice D) are also controversial with some believing that a high fat intake is associated with a greater risk of breast cancer. The patient in this case began menstruating after her sister and began menopause before her. She eats a low fat diet, and does not take hormones. She did, however, have a child at an older age in comparison to her sister.

HRT (choice E) is somewhat controversial, with some studies finding a small increase in the incidence of breast cancer with HRT usage.

 

 

A 24-year-old woman comes to the clinic for a periodic health maintenance examination. She has no complaints. She exercises daily, eats a low fat diet, drinks “a couple of beers” with friends on the weekends, and is a “social” cigarette smoker. She has multiple sexual partners and uses oral contraceptive pills as birth control. She does not use condoms because “it is not as pleasurable.” Her blood pressure is 110/70 mm Hg and pulse is 60/min. Her physical examination is unremarkable. You perform a pelvic examination and send a Pap smear to the laboratory for evaluation. The results, which return 5 days later, show two superficial squamous cells with sharply demarcated, large perinuclear vacuoles and alterations in the chromatin pattern. They use the term “koilocytic atypia.” At this time the most correct statement about her condition is:

  A. Acyclovir will decrease the shedding of the organism
  B. Her Pap smear findings are unrelated to her sexual activity
  C. She and her sexual partners should be treated with metronidazole
  D. She may be at an increased risk for developing cervical cancer
  E. She will develop pelvic inflammatory disease with perihepatitis
Explanation:

The correct answer is D. This patient most likely has an infection with the human papilloma virus (HPV) and depending on the viral subtype (types 16, 18, 31 and 33), she may be at an increased risk for developing cervical carcinoma. Subtypes 6 and 11 are associated with condyloma acuminata which manifests as genital warts. The typical Pap smear findings of a HPV infection include a perinuclear halo and a wrinkled nucleus. HPV infection is associated with cervical dysplasia which may progress to invasive squamous cell carcinoma.

Acyclovir may decrease the shedding of the organism (choice A) if this patient has a herpes infection. A Pap smear of a patient with herpes typically shows giant multinucleated cells with eosinophilic intranuclear inclusions. This patient most likely has an HPV infection, not a herpes infection.

Her Pap smear findings are unrelated to her sexual activity (choice B) is incorrect because she most likely has an HPV infection, which is a sexually transmitted disease.

If this patient had a trichomonas infection, she and her sexual partners should be treated with metronidazole (choice C). However, she most likely has an HPV infection. Trichomonas often presents with a frothy vaginal discharge. A wet mount of the discharge will show motile, flagellated organisms.

She will develop pelvic inflammatory disease (PID) with perihepatitis (choice E) is incorrect because this patient most likely has an infection with HPV, which is associated with condyloma acuminata, cervical dysplasia, and cervical carcinoma. PID with perihepatitis (Fitz-Hugh-Curtis syndrome) is typically associated with chlamydia and gonorrhea infections. Chlamydia and gonorrhea infections are associated with cervicitis, which may progress to PID. They are diagnosed with cervical cultures. Chlamydia is treated with doxycycline, erythromycin, or azithromycin. Gonorrhea is treated with ceftriaxone.

 

 

A 19-year-old woman who is 16-weeks pregnant comes to the office for her first prenatal visit. She has a history of 2 prior pregnancies, both of which were terminated by elective abortions. However, she would like to continue this pregnancy. Her pregnancy to date has been uneventful. Upon further questioning, she admits that she has been smoking “crack cocaine” for the past year on and off. She denies being addicted and says “I can stop using it whenever I want”. You encourage her to try to stop using crack cocaine and suggest that she enroll in a drug treatment program. You try to convince her of the harmful effects of all drug use during pregnancy. You should tell her that cocaine increases her risk for

  A. cerebral vein thrombosis
  B. eclampsia
  C. placental abruption
  D. placenta previa
  E. toxic shock syndrome
Explanation:

The correct answer is C. Cocaine use during pregnancy is known to be associated with vascular problems due to its vasoconstricting properties. It is associated with placental abruption, preterm delivery, intrauterine growth retardation, and birth defects involving the skull, limbs, and urogenital system. Placental abruption is the premature separation of the placenta from the uterine wall. The greater the amount of separation, the greater the risk to both mother and fetus. Conditions that increase the risk of abruption include chronic hypertension, smoking, toxemia, and the usage of any form of cocaine. Blunt trauma to the abdomen during pregnancy can also increase the risk of abruption.

A cerebral vein thrombosis (choice A) and a dural venous thrombosis are seen in hypercoagulable states such as pregnancy. There is no increased risk, however, of a cerebral vein thrombosis with the use of cocaine.

Eclampsia (choice B) is the occurrence of one or more seizures in association with the syndrome of preeclampsia. The cause of preeclampsia is not known. Preeclampsia affects at least 5% of all pregnancies and is a rapidly progressive condition characterized by hypertension, edema, and proteinuria. Typically, preeclampsia occurs in the late second or third trimesters. The use of cocaine during pregnancy is not associated with an increased risk of developing preeclampsia or eclampsia.

Placenta previa (choice D) is defined as the implantation of the placenta over or near the internal os of the cervix. The exact etiology of placenta previa is unknown. This abnormal implantation of the placenta is thought to be related to multiple gestations, advanced maternal age, previous cesarean delivery, previous abortion, and possibly smoking. There is no association with cocaine use.

Toxic shock syndrome (choice E) is caused by a highly invasive group A Streptococcus with or without necrotizing fasciitis and is associated with shock and organ failure. There is no increased risk of toxic shock syndrome during pregnancy or with cocaine use. A form of toxic shock syndrome caused by Staphylococcus aureus is a rare illness occurring mostly in menstruating women who use high-absorbency tampons.

 

 

A 32-year-old woman is admitted to the hospital because of severe left-sided abdominal pain and vaginal bleeding for the past 24 hours. She says that her last menstrual period was 7 weeks ago, which is unusual because her menstrual period “always” occurs every 29 days. She states that she may be pregnant, but also says that she has started a new job and has been working long hours lately. She just assumed that her cycle is “adjusting to this new lifestyle.” She is married, does not have any children, and has never been pregnant. Physical examination shows a tender left-sided adnexal mass and blood at the cervix. An ultrasound shows a left-sided adnexal mass. Beta-human chorionic gonadotropin levels are positive, but low for gestational age. Her blood type is O, Rh-negative. A laparoscopy is performed and an ectopic pregnancy is resected. She recovers from the procedure and is scheduled to be discharged in 24 hours. The most appropriate next step in management to

  A. administer betamethasone
  B. administer medroxyprogesterone
  C. administer RhoGAM
  D. infuse type O-negative blood
  E. schedule a dilation and evacuation
Explanation:

The correct answer is C. Although this patient is Rh-negative, RhoGAM is indicated (Rh immunoglobulin) because there is a risk of isoimmunization. During an ectopic pregnancy, the woman may be exposed to fetal red blood cells and develop antibodies to fetal antigens, which can affect subsequent pregnancies. If the mother is Rh-negative and develops antibodies, and the subsequent fetus is Rh-positive, the antibodies may cross the placenta and cause hemolysis of fetal red blood cells. This process can lead to significant fetal morbidity and mortality, including fetal anemia and erythroblastosis fetalis. Isoimmunization can usually be prevented by giving RhoGAM to any Rh-negative pregnant woman who has an episode of bleeding. Other times that an RhoGAM is necessary is following an ectopic pregnancy, a spontaneous or induced abortion, or an amniocentesis. If there is no bleeding during the pregnancy, then it should be given routinely at about 28 weeks and again within three days of delivering a Rh-positive neonate.

Betamethasone (choice A), which is a steroid, is given to patients experiencing preterm labor to enhance pulmonary maturity. An infant born before 32-34 weeks of gestation is at risk of developing respiratory distress syndrome (RDS). Treatment with steroids has been shown to reduce the incidence of RDS and other complications associated with preterm labor and birth.

An injection of medroxyprogesterone (choice B) is inappropriate at this time because the patient has not requested this method of birth control. Medroxyprogesterone, which is also called Depo-Provera, is a long-acting injectable hormonal contraception that is typically given every three months. It is not standard practice to routinely administer medroxyprogesterone following an ectopic pregnancy.

An infusion of type O-negative blood (choice D) is not indicated because this patient is stable and does not require a blood transfusion.

Dilation and evacuation (choice E) is a method of pregnancy termination in which the cervix is gradually dilated and the uterine contents are extracted. This method is used for second trimester therapeutic abortions, intrauterine fetal death, and incomplete and missed abortions. Dilation and evacuation is not used to treat an ectopic pregnancy.

 

 

A 38-year-old woman is brought to the emergency department by ambulance after being found moaning in the middle of the street near a shopping cart with her belongings. She is clutching her abdomen, moaning, and rocking back and forth. She is wearing tattered clothes, is unkempt and disheveled. After partially undressing her in the emergency department, she appears to be pregnant with a large gravid abdomen. Her underwear is stained with approximately 10cc of dark blood. The nurse finds drug paraphernalia in her jacket pocket. Her temperature is 37.3 C (99.1 F), blood pressure is 142/94 mm Hg, pulse is 125/min, and respirations are 26/min. On examination, she has pinpoint pupils and a tetanic (continuous) abdominal contraction. Her fundal height is 34 centimeters. No pelvic examination is performed. On brief ultrasonic evaluation, a fetus is visualized, but no fetal cardiac activity is visualized. A urine toxicology screen is positive for cocaine. Laboratory studies show:

The patient’s vaginal bleeding is most likely caused by

  A. abruptio placenta
  B. bleeding associated with labor and cervical change
  C. vasa previa
  D. placenta previa
  E. preeclampsia
Explanation:

The correct answer is A. The differential diagnosis of bleeding in the third trimester of pregnancy includes placental abruption, placenta previa, vasa previa, and bleeding associated with labor. Abruption refers to separation of the placenta from the uterus, which leads to bleeding and sometimes to an intrauterine fetal demise. This can occur in any pregnancy but is more common in the setting of drug abuse, smoking, and abdominal trauma (falling on the abdomen, domestic violence, motor vehicle accident). The patient is at high risk for an abruption secondary to her cocaine abuse. Although only 10 cc of blood is visible externally, it is possible that 1-2 liters of blood is being sequestered behind the placenta. This would explain her anemia, low platelet count, and her tetanic contraction (a common uterine response to a severe abruption).

Bleeding associated with labor is often called “bloody show” (choice B). It is caused by cervical change and is typically small volume, bright red, and mixed with cervical mucus.

A vasa previa refers to implantation of the fetal umbilical cord into the fetal membranes instead of the placenta (choice C). This condition is extremely difficult to detect prior to labor. When rupture of membranes occurs, the umbilical cord may be severed resulting in fetal exsanguination. This bleeding is bright red and occurs very abruptly.

A placenta previa refers to a placenta that covers the internal cervical os (choice D). Bleeding associated with a placenta previa can be both of small volume or can be catastrophic. Placental abruption and previa are the most common causes for bleeding in the third trimester. Although it is possible that the patient has a previa, her cocaine abuse is more consistent with an abruption.

Preeclampsia is not associated with vaginal bleeding unless an abruption occurs (choice E). The patient’s blood pressure is slightly elevated at 142/94. However, the patient’s recent cocaine use is more likely to be causing her high blood pressure than preeclampsia. She shows no evidence of hemoconcentration or elevated creatinine that can occur with preeclampsia.

 

A primigravid 24-year-old woman at 34-weeks gestation comes to the clinic because of a pruritic rash that has been developing on her abdomen over the last week. She is otherwise well and has had normal prenatal visits and blood work. She lives with her husband who does not appear to be affected. Physical examination shows numerous 1-2 mm erythematous, edematous vesicular papules along the periumbilical striae distensae but sparing the umbilicus. There is an extension of similar lesions to her upper medial thighs. No pustules, bullae, or burrows are appreciated. The remainder of the physical examination is unremarkable. A biochemical profile is normal. She should be told that she has

  A. a form of pustular psoriasis and should be started on systemic corticosteroid
  B. Herpes gestationis and will most likely develop similar lesions in subsequent pregnancies
  C. prurigo gravidarum and will be at increased risk for postpartum hemorrhage
  D. pruritic urticarial papules and plaques of pregnancy and it typically will resolve postpartum without treatment
  E. scabies and needs to be treated with permethrin
Explanation:

The correct answer is D. Pruritic urticarial papules and plaques of pregnancy (PUPPP) is a relatively common, intense pruritic dermatosis of pregnancy that typically occurs in the third trimester in primigravidas. PUPPP typically develops on the abdomen, especially with periumbilical striae distensae, while the umbilicus is usually spared in contrast to patients with Herpes gestationis. PUPPP is characterized by 1-2 mm erythematous papules that coalesce to form urticarial plaques that may involve a large portion of the abdomen. PUPPP usually responds to topical corticosteroid for symptomatic relief and remits postpartum. It does not recur in subsequent pregnancies, flares with ingestion of oral contraceptives, and is not associated with increased fetal morbidity and mortality.

Impetigo herpetiformis, a rare form of pustular psoriasis (choice A) in pregnant females, is marked by acute, febrile onset with development of grouped erythematous plaques that are rimmed with small sterile pustules. Lesional plaques predominate in flexural areas and expand peripherally. It is usually associated with significant constitutional symptoms and hypocalcemia with secondary delirium, seizures, and tetany. It usually promptly remits after delivery but may recur in subsequent pregnancies.

Herpes gestationis (choice B), also known as pemphigoid gestationis, is a rare autoimmune disease that occurs exclusively during the second or third trimester of pregnancy, or during the immediate postpartum period. It is characterized by intense pruritic papules and tense blisters. Classically, Herpes gestationis presents as urticarial eruption on the abdomen that usually evolves into a pemphigoid like eruption, sparing the face, mucous membranes, palms, and soles. The umbilicus is usually involved, in contrast to PUPPP. The disease tends to recur during subsequent gestations and one quarter of women experience flare-ups with use of oral contraceptives.

Prurigo gravidarum (choice C) is a hepatic condition that usually occurs late in pregnancy. The initial cutaneous manifestation is pruritus, which usually precedes onset of jaundice by 2-4 weeks. It tends to remit soon after delivery but typically recurs in subsequent gestations. It may recur in susceptible individuals after exposure to oral contraceptives. Some reports suggested increased incidence of prematurity, stillbirth, and postpartum hemorrhage.

Scabies (choice E) is infestation with the human itch mite Sarcoptes scabiei, which is spread primarily by person-to-person contact. The chief symptom is pruritus, which can occur on any part of the body but most commonly occurs on the hands, waist, and genital area. The most diagnostic lesions are burrows, which are small, crooked lines 4-6 mm in length that are most common in the web spaces of the fingers, sides of hands, flexor surfaces of wrist, and lateral and medial surfaces of the foot nearest the heel. Mainstay of therapy include permethrin 5% and lindane. Ivermectin single oral dose has been reported to be effective for severe scabies.

A 19-year-old college student comes to the student health service because she “desperately needs help.” Her boyfriend is visiting for the weekend and they had unprotected intercourse that morning. She usually uses condoms for contraception, but had run out. Other than exercise-induced asthma, she has no medical problems. The only medication that she uses is an inhaler for her asthma. She does not smoke cigarettes or drink alcohol. She admits to some occasional marijuana use. She saw a gynecologist 5 months ago and reports a normal Pap smear at that time. She has normal menstrual periods every month, and is expecting her menses any day. She wants emergency contraception in order to prevent pregnancy. Prior to giving her the appropriate medication, the most appropriate next step is to

  A. obtain a gonorrhea/chlamydia probe from her endocervix
  B. order a beta HCG test
  C. order serologic testing for HIV
  D. order a urine drug toxicology screen
  E. perform a vaginal wet mount
Explanation:

The correct answer is B. Prior to giving a patient emergency contraception, it is necessary to establish that she is not clinically pregnant. A beta HCG test is the appropriate test to make sure she is not already pregnant.

Unprotected intercourse does put a patient at risk for other sexually transmitted diseases, such as gonorrhea, chlamydia (choice A), trichomonas, and HIV (choice C). It is necessary to counsel the patient about this, and offer her testing. However, this is not a necessity prior to giving emergency contraception.

A urine toxicology screen (choice D) is not necessary prior to administration of emergency contraception. However, a physician may not want to give increased estrogens to a woman who smokes tobacco, as this increases chances for thromboembolic events.

A wet mount (choice E) is useful for detecting trichomonas. It is not necessary to diagnose and treat this before providing emergency contraception.

 

 

You get a call from a pregnant hepatitis B surface antigen positive woman that you have been taking care of. She is frantic because she unexpectedly went into labor and delivered the baby in her bathtub 30 minutes earlier. She had her husband cut the cord with a clean kitchen knife and the baby appears to be doing well. She wants to know if you can see her immediately. She is concerned about how her hepatitis status affects breast-feeding. You tell her to bring the baby over to the office and

  A. not to breast-feed until the baby received the proper immunizations
  B. that she can begin to breast-feed immediately if the baby is hungry
  C. that her newborn will need to be treated with alpha interferon
  D. that her breast milk does not contain hepatitis B surface antigen
  E. that she should not breast-feed the newborn because of her hepatitis status
Explanation:

The correct answer is B. Even though the breast milk from a hepatitis B surface antigen positive woman contains the hepatitis B surface antigen, according to the American Academy of Pediatrics and the Report of the Committee on Infectious Diseases, breast-feeding does not significantly increase the risk of infection. Newborns born to HBsAg positive women should be given the hepatitis B vaccine and hepatitis B immune globulin as soon after birth as possible, but breast-feeding does not need to be delayed until after immunizations. Therefore, she should be advised that she can begin to breast-feed immediately if the baby is hungry.

As stated above, it is not necessary to wait to breast-feed until after the baby is immunized. Therefore, telling her not to breast-feed until after the baby receives the proper immunizations is incorrect (choice A).

Alpha interferon (choice C) is used to induce remission in patients with chronic liver disease. It is not routinely given to newborns born to HBsAg positive women.

It is incorrect to tell her that her breast milk does not contain hepatitis B surface antigen (choice D) because these antigens have been detected in the breast milk of HBsAg positive women.

It is incorrect to tell her that she should not breast-feed the newborn because of her hepatitis status (choice E) because as stated above, HBsAg positivity is not a contraindication for breast-feeding.

 

 

A 23-year-old G1P0 is in the hospital after the delivery of a healthy baby girl 24 hours ago. She had an unassisted vaginal delivery after a prolonged induction of labor at 41-weeks gestational age. The placenta was expelled 10 minutes after delivery and it appeared to be intact. On the morning of the second hospitalization day, the patient reports heavy vaginal bleeding and minimal pain at the midline episiotomy site. Vital signs are: temperature 37.2 C (99.0 F), blood pressure 136/70 mm Hg, and pulse 90/min. Bimanual examination of the pelvis reveals a boggy uterus. The most appropriate initial management of this patient is

  A. hypogastric artery ligation
  B. selective arterial embolization
  C. uterine artery ligation
  D. uterine massage
  E. uterine packing
Explanation:

The correct answer is D. Uterine atony is the most likely etiology of this patient’s postpartum bleeding. The likelihood of uterine atony increases with prolonged labor, difficult delivery, and prolonged pregnancy. Initial management of the patient should include a thorough pelvic examination for lacerations and signs of atony, such as a boggy uterus. Uterine massage should be initiated immediately, if atony is suspected. Occasionally, appropriate uterine contractions are stimulated with uterine massage and oxytocics alone.

Hypogastric artery ligation (choice A) is not the appropriate initial management of this patient. Surgical measures are reserved for those patients who do not respond to medical therapy first.

Selective arterial embolization (choice B) is an interventional radiology procedure which involves an angiogram to localize the specific vessel or vessels that are bleeding. Metallic coils or other materials are then used to embolize these selective vessels. It is not the appropriate initial management of this patient. Invasive procedures are reserved for those patients who do not respond to medical therapy first.

Uterine artery ligation (choice C) is not the appropriate initial management of this patient. Surgical measures are reserved for those patients who do not respond to medical therapy first.

Uterine packing (choice E) is not the most appropriate initial management of this patient. If uterine massage and oxytocics do not adequately stimulate uterine contractions, packing the uterine cavity with gauze can be performed. This is a temporizing measure while the patient waits for more definitive therapy.

 

 

A 32-year-old woman comes to the emergency department with vaginal bleeding and severe bilateral lower quadrant pain. She tells you that she recently found out that she is pregnant and says, “this is my first pregnancy and I’m afraid I’m going to lose it!” She is pale and in obvious pain. Her last menstrual period was approximately 8 weeks ago and she has had irregular prenatal care. Her past medical history is significant for depression, a treated chlamydia infection, and migraine headaches. Vital signs are: temperature 37.0 C (98.6 F), blood pressure 140/90 mm Hg, and pulse 90/min. Pelvic examination reveals dark blood in the vaginal vault and a closed cervical os. A urine pregnancy test is positive and a serum Beta-HCG is pending. The most appropriate next step in management is to

  A. administer methotrexate therapy
  B. arrange outpatient follow up appointment with an obstetrician for better prenatal care
  C. order a CT scan of the pelvis
  D. order an MRI of the pelvis
  E. order ultrasonography of the pelvis
Explanation:

The correct answer is E. The most emergent diagnosis of concern in a pregnant female presenting with pain and vaginal bleeding is an ectopic pregnancy. In particular, a ruptured ectopic pregnancy is a life-threatening condition and requires emergent surgery. The diagnostic study of choice is ultrasonography of the pelvis. Ultrasonography is a specific and quick way to evaluate for an intrauterine pregnancy. The demonstration of a normal intrauterine pregnancy dramatically decreases the likelihood of a concomitant ectopic pregnancy. In fact, the chance of having both occur simultaneously is approximately 1 in 30,000. Sonographic findings that are highly suspicious for ectopic pregnancy include the absence of an intrauterine pregnancy, an adnexal mass, and free fluid in the pelvis. Ectopic pregnancy is also more common in women with a history of pelvic inflammatory disease.

Methotrexate therapy (choice A) is a medical treatment for ectopic pregnancies. In the past, surgery was the only treatment available for ectopic pregnancies, even if the patient was hemodynamically stable. If the ectopic pregnancy is small and the hormone levels aren’t high, there is a high probability that medical management with methotrexate will be sufficient. Although this is a relatively new treatment (it has been used for about 7-8 years), it is one, which has been found to be quite safe and an alternative to surgery in appropriate cases. There are side effects associated with methotrexate and it should not be administered until there is documentation of an ectopic pregnancy by ultrasonography.

Arranging an outpatient follow up appointment with an obstetrician (choice B) addresses the patient’s needs for better prenatal care. It does not, however, address the need to diagnose a potentially life-threatening condition such as a ruptured ectopic pregnancy.

A CT of the pelvis (choice C) is not an appropriate step in the management of a pregnant patient. CT of the body, and particularly of the pelvis, exposes the developing fetus to harmful radiation. Ultrasonography of the pelvis is more specific and sensitive than a CT in the evaluation of a pregnancy without the danger of radiation exposure.

MRI of the pelvis (choice D) is not an appropriate step in the management of this patient. Ultrasonography is the diagnostic tool of choice in the evaluation of intrauterine and potentially ectopic pregnancies. MRI of the pelvis is useful to evaluate masses such ovarian lesions and leiomyomas of the uterus. There is not a lot of information regarding the effects of MRI on the developing fetus at this time.

 

 

A 48-year-old homemaker comes to the office for a periodic health maintenance examination. Although this is the first time she has seen a doctor in several years, she is in good health with a surgical history remarkable for an appendectomy at age15. She has been married for 18 years and has 2 children. She goes to aerobics class 3 times a week, drinks approximately 2 glasses of wine a night, and has smoked about 5 cigarettes a day for 28 years. She still gets regular periods and has no signs or symptoms of menopause. She does state that in the past few weeks she has noted some blood on the toilet paper when she uses the bathroom after intercourse with her husband. On pelvic exam you find a 1.5 cm friable lesion, which appears to be protruding from her internal os. In addition, you suspect a Candida infection from gross inspection. The most appropriate next step is to

  A. arrange for cervical cone biopsy
  B. do colposcopy
  C. perform a routine Pap test and await pathology results before proceeding
  D. take a biopsy of the lesion
  E. treat the yeast infection and ask her to return for reexamination in 1 week
Explanation:

The correct answer is D. A biopsy should always be taken immediately when a cervical lesion is found on physical exam. With the widespread use of the Pap test, most cases of postcoital bleeding are due to benign causes such as cervicitis, cervical eversion, cervical polyps, or prolapsed submucous fibroids. However, cervical cancer must be ruled out. Once cervical cancers grow to sufficient size they often cause postcoital or intermenstrual bleeding and sometimes cause significant hemorrhage.

Cervical cone biopsy (choice A) is not indicated at this point, as the diagnosis can most likely be made with less invasive means.

A colposcopy (choice B) can sometimes be helpful in determining the appropriate area to biopsy. However, when a gross lesion presents, a colposcopy is not necessary to aid in biopsy.

A routine Pap test (choice C) can be performed, but should not be relied upon as the sole diagnostic procedure. Sensitivity of the Pap test in detecting invasive cervical cancer can be as low as 50%. As stated above, a biopsy should be obtained immediately and you should not await the results of the Pap smear.

This patient’s yeast infection should be treated (choice E), but not at the expense of taking a biopsy. The yeast infection will not affect the biopsy results.

 

 

A 27-year-old woman who is 3 weeks postpartum, comes to the clinic because of left breast pain, body aches, and fever. She had a normal vaginal delivery and an uneventful postpartum period. Four days ago, she started feeling feverish with diffuse body aches. She is concerned, because she is breast-feeding her infant and her left breast has also been tender for the last 4 days. She denies cough, sore throat, or rhinorrhea. Vital signs are: temperature 39.3 C (102.7 F), blood pressure 120/65 mm Hg, and pulse 90/min. Breast examination reveals a moderately erythematous left breast with diffuse tenderness. The right breast is normal. The rest of the physical examination is normal. The most appropriate management of this patient is to

  A. advise the patient to discontinue breast-feeding from the left breast
  B. culture her breast milk
  C. order an ultrasound of the left breast
  D. prescribe oral penicillinase-resistant antibiotic
  E. refer her for a surgical consult for abscess drainage
Explanation:

The correct answer is D. The symptoms of fever, breast erythema and tenderness, and general body aches in the postpartum period is most consistent with mastitis. This breast infection occurs most commonly several weeks after delivery. The infection is usually unilateral and is thought to be transmitted from the infant’s pharynx. Staphylococcus aureus is the most common organism, and therefore, the recommended treatment is a penicillinase-resistant antibiotic such as dicloxacillin.

It is not necessary for the mother to discontinue breast-feeding from the left breast (choice A) in the setting of mastitis. The route of infection is thought to originate in the infant’s pharynx and there should be no concern of transmitting the infection to the infant.

Culturing the breast milk (choice B) is no longer routinely done. Prior studies of breast milk cultures have demonstrated the prevalence of Staphylococcus aureus in 50% of cases. No other single predominant organism was isolated in the other 50%. Based on this data, it is appropriate to treat mastitis with penicillinase-resistant antibiotics without a culture of the breast milk.

Ordering an ultrasound of the breast (choice C) is not necessary. The diagnosis of mastitis is made clinically.

A surgical consult (choice E) is not the appropriate management of mastitis. This patient is clearly presenting with signs and symptoms of mastitis and there is no clinical indication of a breast abscess.

 

 

A 36-year-old woman comes to the office for evaluation of abnormal discharge from her right eye. She had seen you for her annual health maintenance examinations and had a Pap smear done just 2 weeks prior. After diagnosing conjunctivitis and giving her the appropriate treatment, you look to see if her Pap smear results are back yet. You notice that the smear had no cytologic abnormalities, but that there were no endocervical cells present on the smear. She is a non-smoker, has yearly Pap tests that have never been abnormal, and has been in a monogamous relationship for 12 years. The most appropriate next step is to

  A. advise her that she needs increased monitoring, and that you will perform a repeat Pap smear in 6 months
  B. advise her that since she is at very low risk for an abnormality, she can wait until her annual exam next year for a re-sample of her endocervix
  C. explain to her that although the reading of the exocervix is valid, you need to re-sample her endocervix
  D. explain to her that she needs to repeat the Pap test, as these results are not valid
  E. perform a colposcopic examination of her cervix
Explanation:

The correct answer is B. The presence of endocervical cells on a Pap test is regarded as evidence of adequate sampling of the transformation zone during cytologic screening of the cervix. When these cells are absent, it indicates that this area may not have been sampled. This is considered a satisfactory, but limited smear. In patients with no known risk factors (i.e., prior abnormal Pap test, multiple sexual partners, smoking) the American College of Obstetricians and Gynecologists recommends that the physician may defer to repeating the Pap test in 12 months.

It is inappropriate to tell her that she needs increased monitoring (choice A) because these results do not indicate any abnormality. It is simply sampling error.

If this were a high risk patient (i.e., prior abnormal Pap test, multiple sexual partners, smoking), a re-sampling only of the endocervix (choice C) only would be the appropriate next step. However, since she is not at high risk, you can wait for 1 year.

It is inappropriate to tell her that the results of this test are not valid (choice D) because they are valid, just not complete.

A colposcopic examination of the cervix (choice E) would be appropriate if this patient had abnormal cells on her smear indicating a pre-cancerous lesion.

 

 

A 3-day-old male infant in the neonatal unit has bilious vomiting for 24 hours. The child is inconsolable and will not feed. Vital signs are: temperature 38 C (100.4 F), pulse 110/min, blood pressure 80/50 mmHg, and respirations 20/min. Abdominal examination is unremarkable. A barium enema demonstrates the cecum to be in the left upper quadrant. There is no right lower quadrant mass on abdominal x-ray. Intravenous antibiotics and Ringer’s solution are administered. The next step in treating this patient is

  A. bowel rest
  B. intussusception reduction with air
  C. intussusception reduction with contrast
  D. laparotomy
  E. repeat barium enema in 24 hours
Explanation:

The correct answer is D. Bilious vomiting in an infant means that there is a malrotation with volvulus until proven otherwise. The radiographic findings of the cecum in the left upper quadrant confirms this clinical diagnosis. An upper gastrointestinal series would likely show a bird-beak deformity of the midgut where there is volvulus of the gut around a mesenteric “Ladd” band. About 20% of malrotation with volvulus is associated with duodenal atresia, annular pancreas, or a duodenal diaphragm. About 33% of cases present in the first week of life and 85% present in the first year of life. The hallmark of malrotation is ischemia of the midgut as the superior mesenteric vein and the superior mesenteric artery are occluded by the twisting mesentery. Immediate surgery is necessary to prevent death or the loss of much of the bowel.

Bowel rest (choice A) would lead to high morbidity and possibly mortality in this patient with midgut malrotation with volvulus. Untreated midgut malrotation with volvulus will lead to mesenteric ischemia.

The diagnosis of intussusception is made with either ultrasound or an enema using either air or contrast medium (choice B and C). This patient’s presentation of bilious vomiting favors the diagnosis of malrotation with volvulus rather than intussusception. Intussusception typically presents in infants from 4 months to 2 years of age, with alternating lethargy and irritability, colicky abdominal pain, and currant jelly stools. In addition, there is usually a right lower quadrant mass on abdominal x-ray.

Repeating the barium enema in 24 hours (choice E) is not appropriate as this will only delay definitive surgical treatment. Repeating this study will not contribute to this patient’s management.

 

 

A 31-year-old woman comes to the emergency department with midline abdominal pain that she reports is 10 out of 10 in severity on a pain scale, with 10 being the worst pain she has ever felt. The pain has been present for 1 hour. She has no past medical history and her last menstrual period was 1 day ago. Her only medication is oral contraceptive pills. She has no allergies. She had an uncomplicated pregnancy 3 years ago and had an uneventful normal vaginal delivery. Vital signs are: temperature 37 C (98.6 F), pulse 90/min, blood pressure 100/70 mm Hg, and respirations 15/min. Oxygen saturation is 96% on room air. Physical examination reveals a woman in obvious pain. There is left adnexal and midline pelvic pain on palpation. A urine pregnancy test is negative. Ultrasonography of the pelvis demonstrates an enlarged left ovary with decreased blood flow. The most likely complication of this finding is

  A. fibroid degeneration
  B. infertility
  C. ovarian cancer
  D. ovarian cysts
  E. pelvic inflammatory disease
Explanation:

The correct answer is B. A young woman with acute pelvic pain localizing to one side is suspicious for ovarian torsion. Associated symptoms may include nausea and vomiting. The diagnosis was confirmed with endovaginal ultrasound demonstrating an edematous ovary with decreased venous and possibly, arterial flow. Complications of ovarian torsion include infection, peritonitis, sepsis, adhesions, chronic pelvic pain, and infertility due to the loss of the viability of the torsed ovary.

Fibroids (choice A) may degenerate over time as they outgrow their blood supply. Moreover, adventitial fibroids may be pedunculated and torse on their stalk. Pain from a degenerating fibroid is rare. Degenerating fibroids have no association with ovarian torsion.

Ovarian cancer (choice C) is not a complication of torsion. The typical presentation of ovarian cancer is diffuse carcinomatosis and abdominal mass or bowel obstruction. Ovarian cancers can enlarge the ovary and may predispose it to twisting on its vascular supply.

Ovarian cysts (choice D) are not a complication of torsion. Benign functional ovarian cysts can get quite large and may predispose a patient to torsion by causing the ovary to twist on its vascular supply.

Pelvic inflammatory disease (choice E) is not a complication of torsion. It is caused by organisms ascending to the upper female genital tract from the vagina and cervix. It most commonly is associated with Chlamydia trachomatis and Neisseria gonorrhoeae, but other organisms, and in many cases multiple organisms, have been isolated.

 

 

A 27-year-old female police officer comes into the emergency department where you are working. She is complaining of a 2-day history of nausea, and vomiting and some mild left lower quadrant pain. Her vital signs are stable. Physical examination shows mild tenderness to palpation in the left lower quadrant. Prior to performing the pelvic examination, the patient informs you that she is menstruating. The examination is significant only for blood in the vault. Routine laboratory studies are sent and a beta hCG returns as 2700. You order a pelvic ultrasound, which identifies nothing in the uterus. However, there is a fetal pole in the left fallopian tube. You inform the patient that she has an ectopic pregnancy and discuss the option of surgery versus methotrexate therapy. The patient decides that she would like to try methotrexate. Appropriate follow-up care for this patient will include

  A. another dose of methotrexate on day 4 of therapy if blood levels of hCG have not fallen by 15%
  B. checking beta HCG on day 3 and 7 of methotrexate therapy
  C. immediate laparotomy if methotrexate therapy is found to fail by day 7 of treatment
  D. repeating beta hCG 2 weeks after diagnosis if beta hCG is found to have fallen greater than 15% from day 4 to day 7 of treatment
  E. repeating pelvic ultrasound in 1 week
Explanation:

The correct answer is D. Ectopic pregnancy can be managed medically in specific situations dependent on size of the mass and whether or not it has ruptured. Treatment with methotrexate is calculated according to body surface area. Patients are counseled extensively on the side effects of methotrexate and given strict ectopic precautions for cases of rupture. Beta hCG is routinely drawn on day 4 and day 7 of treatment. If the beta hCG drops 15% from day 4 to day 7, treatment is thought to be successful and the patient should return weekly for beta hCG blood draws until it is negative.

Beta hCG peaks on day 4 of treatment. It is on day 7 of treatment that another dose of methotrexate may be given if levels do not fall 15% from day 4 of treatment. Another dose of methotrexate on day 4 of therapy if blood levels of hCG have not fallen by 15% (choice A) is incorrect.

Levels are routinely checked on day 4 and day 7 of treatment, not 3 and 7 (choice B).

If methotrexate therapy is found to have failed by day 7 of therapy another dose of methotrexate can be tried. Immediate laparotomy (choice C) is indicated in cases of rupture.

Pelvic ultrasound (choice E) does not need to be repeated unless indicated by a change in the patient’s clinical status.

 

 

A 26-year-old woman comes to the clinic because of a 3-day history of a “bad smelling”, grayish colored vaginal discharge. She says that she has had 4 different sexual partners in the past 5 years, and she has never had a sexually transmitted disease. Her current sexual partner is asymptomatic. Physical examination is unremarkable. Pelvic examination shows a moderate amount of gray vaginal discharge at the introitus and adherent to the vaginal walls. The pH of the discharge is 5.1. There is a fishy odor released when you expose the discharge to KOH. You decide to do a wet-mount preparation of the discharge. Based on the history and pelvic exam, you expect to see

  A. epithelial cells with smudged borders
  B. Gram-negative intracellular diplococci
  C. motile flagellated organisms
  D. no cells; a culture would be required for diagnosis
  E. yeast and pseudohyphae
Explanation:

The correct answer is A. Epithelial cells with smudged borders are often called “clue” cells and are due to bacteria adherent to the cell membrane which is characteristic of bacterial vaginosis (BV). BV is a syndrome seen in sexually active females which may be asymptomatic but often presents with a thin, white-gray, foul smelling discharge which people often describe as “fishy”. BV is usually not associated with pruritus, dysuria, or abdominal pain. BV is the most prevalent vaginal infection in sexually active females. Although not completely clear, causes are thought to include Gardnerella vaginalis, Mycoplasma hominis, and anaerobic bacteria. In order to diagnose BV, a woman must have at least 3 of the following symptoms, including a white-gray non-inflammatory vaginal discharge that adheres to the vaginal wall, vaginal fluid pH greater than 4.5, a “fishy” odor to the vaginal fluid either before or after mixing with 10% potassium hydroxide, and the presence of clue cells. Metronidazole is the treatment. Treatment of the sexual partner is not routinely recommended.

Gram-negative intracellular diplococci (choice B) are indicative of a gonorrheal infection and would be seen on a Gram stain, not a wet mount, which is prepared with saline. The infection may be asymptomatic, however it may be associated with a malodorous, purulent (yellowish-green) vaginal discharge. A culture on a Thayer-Martin agar plate is often used to establish the diagnosis. Enzyme immunoassays are also available. The treatment includes ceftriaxone and doxycycline (because of the high incidence of co-infection with chlamydia). Treatment of the sexual partner is routinely recommended.

Motile flagellated organisms (choice C) are seen in Trichomonas vaginalis. Although the infection is often asymptomatic, men may develop urethritis, and women may complain of a frothy vaginal discharge that is green or yellow in color and may have a musky odor. Women may also have some lower abdominal tenderness with more severe symptoms just before or just after menstruation. On exam, the vaginal mucosa may be erythematous with an inflamed and friable cervix. Women may also have “strawberry cervix,” a term used when there are multiple petechiae on the cervix. A wet prep will often show trichomonads with their flagella and jerky motility. Metronidazole is the treatment. Simultaneous treatment of the partner is necessary.

BV can be diagnosed by finding a white-gray non-inflammatory vaginal discharge that adheres to the vaginal wall, a pH > 4.5, an amine odor when combined with KOH, and clue cells on a wet mount. A culture is not necessary to establish the diagnosis (choice D).

In a woman with a yeast infection, you would expect to see yeast and pseudohyphae (choice E) on a wet mount. These women usually present with complaints of intense pruritus and burning accompanied by a thick white vaginal discharge. Yeast infections are caused by overgrowth of Candida albicans often due to factors such as pregnancy, antibiotic use, diabetes, and oral contraceptive use. Some women report predisposition to yeast infections immediately preceding menstruation. The treatment includes imidazole creams or suppositories. Treatment of the asymptomatic sexual partner is not routinely recommended.

 

 

A 31-year-old woman comes to the clinic because of “not having a period for over a year, white discharge from both nipples, and severe frontal headaches of 2 years duration”. Evaluation reveals a follicle-stimulating hormone (FSH) level of 6.0 mIU/mL, basal serum prolactin level of 82 ng/mL, and thyroid-stimulating hormone (TSH) of 19 micro-U/mL (normal ranges: FSH 2-20 mIU/mL, prolactin, <20 ng/mL, TSH, 0.5-5.0 microU/mL). An MRI reveals pituitary enlargement with a mass measuring 13 mm in diameter. The next step in the management of this patient is

  A. evaluation of other pituitary hormones
  B. formal visual field testing
  C. referral to a neurosurgeon
  D. therapy with bromocriptine
  E. therapy with levothyroxine
Explanation:

The correct answer is E. Prolactin, unlike other anterior pituitary hormones, is primarily under tonic inhibition by dopamine secreted from the hypothalamus. Prolactin release can be stimulated by many factors including thyrotropin-releasing hormone (TRH). Because TRH is increased in primary hypothyroidism, the circulating TSH (not TRH) should be measured in all women who have amenorrhea or galactorrhea, or both, to exclude hypothyroidism. Increased TSH levels should be presumed to result from primary hypothyroidism. Many reports have documented that primary hypothyroidism can mimic a pituitary tumor and can lead to profound pituitary enlargement, because of the hypertrophy of thyrotrophs. After treatment with levothyroxine, the enlargement of the pituitary should subside and the patient should become euthyroid.

Evaluation of other pituitary hormones (choice A) is not indicated at this time. Evaluation of other pituitary hormones not measured initially such as LH, and GH is not necessary since the elevation of TSH is diagnostic for hypothyroidism.

Formal visual field testing (choice B) is not indicated at this time. Formal visual field testing would be important if you suspected bitemporal hemianopia (caused by a large tumor in the area of the sella turcica putting pressure on the optic chiasm).

Referral to a neurosurgeon (choice C) is not appropriate at this time. You should refer her to a neurosurgeon only after establishing that a tumor, or a macroadenoma, is the cause of enlargement in the pituitary, and you have treated the patient with levothyroxine without improvement in the size of the pituitary and functional status of the thyroid.

Therapy with bromocriptine (choice D) is incorrect. Since bromocriptine is a dopamine receptor agonist, it would decrease prolactin. This may be useful in hyperprolactinemic states, but not in hypothyroidism.

 

 

You are called to see a patient on the postpartum floor of a small community hospital. She is a 24-year-old woman who is 2 days postpartum after a normal spontaneous vaginal delivery of a healthy baby boy. This is her first child. She has no medical problems, had a pregnancy complicated only by some early nausea and vomiting, and has never had surgery before. She was compliant with taking prenatal vitamins and iron throughout pregnancy and has continued taking them since her delivery. She has been trying to breastfeed her son, as she understands that it is beneficial to bonding and to the overall health of the newborn. She has good milk letdown. Her temperature is 37.2 C (99.0 F) and she is complaining of pain in both breasts. The only remarkable finding on physical examination is firm, tender breasts. The most appropriate management at this time is to

  A. advise her to bottle feed only until the fever subsides without using the breast pump
  B. advise her to feed more frequently in addition to pumping the breasts as needed
  C. advise her to use breast pump only and use this milk for baby feeds
  D. apply cold compresses to breasts while continuing to breast feed
  E. begin dicloxacillin therapy while continuing to breast feed
Explanation:

The correct answer is B. Breast engorgement is a common problem found in postpartum women. It is characterized by slight fever and bilateral rock-hard breasts. Clogged milk ducts can present as localized tender masses. If the woman is breast feeding, she should be advised to empty her breasts more often, and is thus encouraged to either pump or breastfeed with increased frequency. Warm compresses are also advisable to enhance milk letdown. If the woman is not breast feeding, she should use cold compresses in addition to a breast binder.

As explained above, emptying the breasts is what decreases the chances of engorgement. Thus, it is imperative that the woman continues to breastfeed or use the breast pump. Therefore, advise her to bottle feed only until the fever subsides without using the breast pump (choice A) is incorrect.

There is no reason that the patient cannot have the baby latch on. Even in the scenario of sore nipples, breastfeeding with latch-on is advocated. She should not be advised to only use the breast pump (choice C).

As explained above, warm compresses facilitate milk letdown, whereas cold compresses (choice D) are utilized for women who are not breast feeding.

The main differential diagnosis for engorgement is mastitis, which occurs in approximately 2% of breastfeeding women, most commonly occurring between the first and fifth weeks postpartum. It is manifested by a sore, reddened area on the breast. The patient will feel ill, with high fevers, chills, and malaise. Approximately 40% of cases are caused by Staphylococcus aureus and the first choice antibiotic is dicloxacillin (choice E). The patient is encouraged to breast feed while being treated.

 

 

An 18-year-old girl comes to the emergency department with her mother because of a 12-hour history of lower abdominal pain and nausea. She is sexually active with 3 different partners and she “usually” uses condoms for contraception. She is unsure of the exact date of her last menstrual period. She has never had a sexually transmitted disease in the past. Her temperature is 38.3 C (101.0 F), blood pressure is 110/70 mm Hg, and pulse is 65/min. Physical examination shows bilateral lower abdominal tenderness, but rebound tenderness and guarding are absent. Pelvic examination shows mild cervical motion tenderness and adnexal tenderness. A small amount of cervical discharge is present. There are no palpable masses. Cervical cultures are taken and sent to pathology for evaluation. The most appropriate next step is to

  A. admit her to the hospital and begin cefoxitin and doxycycline therapy, intravenously
  B. do a culdocentesis
  C. do urinary B– human chorionic gonadotropin testing
  D. prescribe ofloxacin and metronidazole therapy, orally, and discharge her
  E. try to contact her male sexual partners for evaluation and treatment
Explanation:

The correct answer is C. It is likely that this patient has either an ectopic pregnancy or pelvic inflammatory disease. The cervical motion tenderness and bilateral pain may be more consistent with PID, which is often characterized by those symptoms and adnexal tenderness, a vaginal or cervical discharge, a fever >38.3 C, (101.0 F), elevated erythrocyte sedimentation rate and a C-reactive protein, and a laboratory documentation of N. gonorrhoeae or C. trachomatis. An ectopic pregnancy usually presents with abdominal pain, abnormal bleeding, and amenorrhea. A pregnancy test should be the next test to rule out an ectopic pregnancy. Also, even though her symptoms are more consistent with PID, because she is not sure of her LMP date, a pregnancy test is indicated at this time.

A pregnancy test should be performed to rule out an ectopic pregnancy before treatment for suspected PID is given. Also, admission and intravenous therapy (choice A) for all adolescents with PID used to be recommended, but this is no longer true because there is not a lot of data to support this recommendation. However, admission is recommended for the following individuals, including noncompliant patients, pregnant patients with PID, an abscess or peritonitis, HIV patients with PID, and all those that fail outpatient therapy.

A culdocentesis (choice B) is not the most appropriate next step at this time. This procedure, which is transvaginal sampling of the cul-de-sac, may be performed to support the diagnosis of an ectopic pregnancy if clotted blood is found. It may also be used to obtain bacteriologic samples. It is not indicated at this time because there is a cervical discharge present that is sent to pathology for evaluation, and a urine pregnancy test must first be done to evaluate for pregnancy.

If this patient has an uncomplicated case of PID (without a tuboovarian abscess), prescribing ofloxacin and metronidazole therapy, orally, and discharging her (choice D) will be the most likely treatment plan. A pregnancy test must first be performed to rule out an ectopic pregnancy.

If this patient has PID, it is appropriate to evaluate and treat sexual partners (choice E). However, since this patient is unsure of the date of her last LMP, a pregnancy test is the most appropriate next step. Contacting sexual partners should wait until therapy for this patient is initiated.

 

 

You are called to the delivery room after a full-term male infant is born via cesarean section to a G2P1 mother. Under the radiant warmer, the baby is crying and has a heart rate of 90/min. There is some flexion of the extremities and he grimaces when the catheter is passed in the nostril. The baby’s body is pink, but the extremities are blue. The baby’s Apgar score at 1 minute is

  A. 5
  B. 6
  C. 7
  D. 8
  E. 9
Explanation:

The correct answer is B. APGAR is a named after Virginia Apgar, M.D. who came up with a rating system for neonates at delivery. One helpful mnemonic is A: appearance, P: pulse, G: grimace or response to catheter in nostril, A: activity or tone, and R: respiratory effort. Each category is rated from 0-2. This child has an apgar of 6.

The remainder of the choices are incorrect as you can see from the chart.

 

 

A 21-year-old woman comes to the student health clinic complaining of “painful periods” for the past few years. She describes the pain as moderate to severe, crampy in nature, and located in her lower abdomen. She has never been sexually active because, as she tells you, she is waiting until she is married. The patient’s physical and pelvic examinations are normal as is a Pap smear, which returns 5 days later. The most appropriate next step in the management of this patient is to

  A. do a progesterone withdrawal test
  B. measure β-Human Chorionic Gonadotropin (β-HCG) levels
  C. perform a dilatation and curettage
  D. recommend a therapeutic trial of ibuprofen
  E. refer her to a psychiatrist
  F. send her for a diagnostic laparoscopy
Explanation:

The correct answer is D. This patient is presenting with classic symptoms of dysmenorrhea, perhaps the most common complaint of gynecologic patients. The pain may precede menses, but typically has its onset with the first day of flow, and subsides after few days. Most women report relief of symptoms with antiprostaglandin medications such as ibuprofen. Heating pads and hot water bottles applied to the lower abdomen are sometimes helpful as well.

The progesterone withdrawal test (choice A) is used in the evaluation of amenorrhea. It has no role in the diagnosis or treatment of dysmenorrhea.

There is no indication that this patient may be pregnant (choice B). She is reporting normal menstrual cycles. Measuring a β-HCG level at this time would not be informative and would not affect the treatment of her dysmenorrhea.

Dilatation and curettage (choice C) is not the most appropriate next step for this patient. This is a procedure that is most commonly used in women with postmenopausal bleeding. It has no role in the treatment of dysmenorrhea.

Referral to a psychiatrist (choice E) is not appropriate at this time. The patient should be started on a therapeutic trial of ibuprofen initially. If the patient continues to complain of pain out of proportion to her physical exam and other diagnostic findings, a psychiatric referral can be considered.

A diagnostic laparoscopy (choice F) is not the most appropriate next step for this patient. It is a procedure that can be used to directly visualize ectopic endometrial tissue, if endometriosis is suspected. Other symptoms associated with endometriosis include dyspareunia and pain with defecation. Physical exam often reveals multiple tender nodules along the uterosacral ligament during a rectal-vaginal examination.

 

A 20-year-old woman comes to the office with a 4-month history of “missed periods.” Prior to these past 4 months she says that she had normal menses, which began when she was 14 years old and occurred every 29 days. She has had “occasional” sexual experiences over the past few years and admits to a total of 4 sexual partners. She and her partners always use condoms for contraception. She recently started a new job that is “stressing her out completely” and her sleep habits have been “a mess.” Her temperature is 37.0 C (98.6 F), blood pressure is 110/70 mm Hg, pulse is 73/min, and respirations are 11/min. Her physical examination, including a pelvic examination, is unremarkable. The most appropriate next step is to

  A. determine beta hCG level
  B. obtain cortisol level
  C. obtain FSH level
  D. order prolactin level
  E. order testosterone level
  F. reassure her that it is most likely due to stressors in her life
Explanation:

The correct answer is A. The most common cause of amenorrhea in women of childbearing age is pregnancy. Despite the fact that this patient she “always” uses condoms during sexual activity, the most appropriate first test would be a serum beta hCG to rule out pregnancy. Elevated levels of cortisol (choice B) can suppress gonadotropins and therefore glucocorticoid excess may be a cause of amenorrhea. It is appropriate to evaluate for Cushing’s syndrome if there are any signs of cortisol excess. Adrenal insufficiency may also cause amenorrhea and a cosyntropin stimulation test may be performed if clinically indicated by signs of adrenal insufficiency such as hypotension, hyperkalemia, hyponatremia, etc.

FSH levels (choice C) are helpful in the evaluation of amenorrhea. A normal FSH suggests a uterine etiology. A low FSH suggests a pituitary etiology for amenorrhea. A high FSH suggests ovarian failure as the etiology of amenorrhea.

Prolactin (choice D) elevations are a common cause of amenorrhea. It should be part of the secondary evaluation of amenorrhea if pregnancy tests are negative. If levels are elevated, an MRI of the pituitary should follow.

Elevated testosterone (choice E) may cause amenorrhea and is most typically elevated in polycystic ovary syndrome. Testing is indicated if amenorrhea is accompanied by virilization.

While stress and lifestyle changes can cause abnormalities in the menstrual cycle (choice F), 4 months of amenorrhea should be evaluated with a pregnancy test since the most common cause of amenorrhea in a woman of childbearing age is pregnancy.

 

 

A 22-year-old pregnant woman is admitted to the hospital for evaluation of severe headaches. She is 20-weeks pregnant and has had a progressively worsening nonpulsatile headache for the past 4 days. Vital signs at the time of admission are: temperature 37.0 C (98.6 F), blood pressure 120/67 mm Hg, and pulse 88/min. Physical examination is normal. A few hours after admission, the patient has a witnessed seizure in which she has several minutes of abnormal tonic-clonic movements. A CT scan of the head is performed and demonstrates cerebral edema within the right temporal lobe and deep gray matter. In addition, there is hyperattenuation within the region of the superior sagittal sinus. A small foci of petechial hemorrhage is also seen in the right temporal lobe. The most appropriate management of this patient is to

  A. administer heparin
  B. administer warfarin
  C. induce labor
  D. order an MRI of the brain
  E. perform a lumbar puncture for CSF analysis
Explanation:

The correct answer is A. Patients with dural sinus and cerebral venous thrombosis can present with headaches, stroke-like symptoms, and/or seizures. There are multiple etiologies including dehydration, sepsis, and trauma. In addition, hypercoagulable states such as sickle cell disease, leukemia, and pregnancy are also predisposing factors. Because the symptomatology of a dural sinus thrombosis is nonspecific, imaging studies play a critical role in the diagnosis. On non-contrast enhanced head CT, a cord sign or tubular hyperdensity may be seen in the acute setting along with cortical and subcortical hemorrhage due to venous stasis and infarction. The treatment of choice during pregnancy is intravenous heparin. Unfractionated heparin and low molecular weight heparin (LMWH) do not cross the placenta and are considered safe for the fetus, but must be administered parenterally.

The administration of warfarin (choice B) is contraindicated during pregnancy. Warfarin crosses the placenta and can cause bleeding and birth defects. Heparin is the anticoagulant of choice during pregnancy because it does not cross the placenta.

The induction of labor (choice C) is not indicated in this patient. She is only 20-weeks pregnant and is not ready to deliver. Furthermore, induction of labor is not necessary in the treatment of dural sinus and cerebral venous thrombosis.

An MRI of the brain (choice D) is not necessary to make the diagnosis of dural sinus thrombosis in this patient. The CT scan of the head already demonstrates findings consistent with thrombosis of the superior sagittal sinus as well as hemorrhage due to venous stasis. If the CT scan of the head is non-diagnostic, an MRI of the brain or a cerebral angiography can be performed for further evaluation.

A lumbar puncture for CSF analysis (choice E) is not necessary for this patient’s diagnosis or treatment. The symptoms and CT findings are consistent with dural sinus thrombosis. CSF analysis is useful in patients who are suspected of having subarachnoid hemorrhage or meningitis. It has no role in dural sinus thrombosis.

 

 

A previously healthy 3-week-old baby is brought by his parents to your emergency department with a 1-day history of emesis. The parents describe the emesis as “forceful”, non-bloody, and non-bilious. The baby is exclusively breastfed and continues to be hungry after each episode of vomiting. They deny any fevers. You notice an active baby boy with unremarkable vital signs. Physical examination is significant for a peristaltic wave on the abdomen and a 2×2 cm firm mass palpated in the midepigastric region. Laboratory studies show a bicarbonate level of 18 mEq/L. The most likely diagnosis is

  A. annular pancreas
  B. gastroesophageal reflux
  C. intussusception
  D. malrotation with volvulus
  E. pyloric stenosis
Explanation:

The correct answer is E. This patient presents with a classic case of pyloric stenosis. It is the most common surgical condition seen in the newborn period. It is more common in males than females. It is caused by a hypertrophic pyloric muscle, which causes obstruction of the gastric outlet. This hypertrophic muscle can be palpated in the midepigastric region and represents the classic “olive” associated with pyloric stenosis. The emesis is non-bilious because the obstruction is proximal to the ligament of Treitz. Once the diagnosis is made, treatment consists of a pyloromyotomy.

Annular pancreas (choice A) is caused by the failure of the resorption of the left pancreatic bud during rotation of the duodenum during fetal development, which causes a ring-like structure around the duodenum, often the second portion. These patients present with signs of partial or complete small bowel obstruction and will have frequent bilious vomiting and abdominal distension often in the first week of life. There is often a history of polyhydramnios.

Patients with gastroesophageal reflux (choice B) rarely present with projectile vomiting. Parents often complain of spitting up with feeds, arching of the back, and waking up from sleep crying. Mild GER often responds to changing to an upright feeding position and thickening the feeds with cereal. Moderate GER may require medication, such as an H2-blocker, for control. Severe reflux may require surgical intervention.

Patients with intussusception (choice C), caused by an invagination of the bowel, classically present with colicky abdominal pain, raising of their knees to their chest to relieve this pain, a sausage-like abdominal mass, and “current jelly” stools. They may also have vomiting that begins as non-bilious and progresses to bilious as the obstruction progresses.

Malrotation with volvulus (choice D) is caused by failure of the small intestine and right colon to reach their proper anatomic location during fetal development and twisting of the gut, resulting in compromised perfusion to this area. These patients have bilious vomiting, may have bloody stools, and appear quite ill as compromised perfusion progresses to infarction and gangrene.

 

 

A 57-year-old gravida 3, para 3 woman comes to the emergency department with abdominal discomfort and says she feels like she is “bloated”. She denies any nausea or vomiting and has had regular bowel movements, but has lost over 15 lbs over the last year unintentionally. Her past medical history and surgical history are unremarkable. Her family history is significant for diabetes and colon cancer. She has smoked half a pack of cigarettes a day for over 20 years, but denies alcohol or drug use. Her vital signs are: temperature of 37.0 C (98.6 F), blood pressure of 137/76 mm Hg, and pulse of 83/min. Physical examination shows abdominal distension and diffuse abdominal pain, but no rebound tenderness or guarding. All of her laboratory studies are within normal range. A transvaginal ultrasound shows a complex left adnexal mass with a solid and cystic component measuring 4 cm by 4 cm in diameter. Besides the appearance of the mass, the other feature that would be helpful in detecting an early malignant ovarian tumor is

  A. differences in temperature of tumor tissue
  B. discordance of ovarian artery blood supply from left to right ovaries
  C. increased blood flow of ovarian arteries
  D. neovascularity of tumor blood supply
  E. ultrasonographic pattern of tissues surrounding the ovaries
Explanation:

The correct answer is D. The most promising technique for ovarian cancer screening and an important adjunct for radiologists in establishing a mass as possibly malignant, is through the use of transvaginal ultrasound (TV U/S) with Doppler color flow imaging. Angiogenesis, the formation of new blood supply, is essential in any tissue. In order for the growing carcinoma to survive, new microvessels have to be established to provide oxygen and nutrients, and to remove waste products. TV U/S on Doppler mode can detect blood movement, “ectopic blood flow patterns,” in these newly formed vessels and can help to detect ovarian tumors at early stages.

Differences in temperature of tumor tissue (choice A) is incorrect. There are no significant differences in temperature between malignant and benign tissues.

Discordance of ovarian artery blood supply from left to right ovaries (choice B) is incorrect because differences of blood flow from one side to the other are very unreliable and certainly not generally useful.

Increased blood flow of ovarian arteries (choice C) is incorrect. Blood flow patterns of ovarian arteries differ from one person to the next. Additionally, increased blood flow of the ovarian arteries feeding a carcinoma are not necessarily true, especially in an early malignant tumor.

The ultrasonographic pattern of tissues surrounding the ovaries (choice E) is incorrect because a tumor would have to be very large, and thus not in its early stage, in order to distort the surrounding tissues. Appearance of the surrounding tissue is generally not useful in predicting malignant versus benign tissue.

 

 

A 37-year-old woman comes to the clinic because of severe headaches. She is 38-weeks pregnant and has had headaches on and off throughout her pregnancy. In the past week the headaches have progressively worsened. Vital signs are: temperature 37.0 C (98.6 F), pulse 70/min, and blood pressure 180/100 mm Hg. Physical examination reveals moderate pitting edema in both lower extremities. Neurologic examination is normal. Fetal heart rate monitoring is performed in the office and it demonstrates a baseline heart rate of 120/min with variable decelerations to 70-80/min. The most appropriate management at this time is to

  A. admit her to hospital for induction of labor
  B. do a complete fetal sonographic survey
  C. order a CT scan of the head
  D. order serum liver function tests and a complete blood count
  E. order a urinalysis to evaluate for protein
Explanation:

The correct answer is A. This patient has signs and symptoms of preeclampsia, which is a syndrome of high blood pressure during pregnancy associated with proteinuria. The complications of preeclampsia include placental abruption, premature labor, and fetal distress. Fetal heart tracing in this case, demonstrates variable decelerations which indicate fetal distress. Giving birth is the only cure for preeclampsia. Factors that influence the decision to induce labor or perform a cesarean delivery, include fetal distress, age of the fetus, and severity of the condition.

A fetal sonographic survey (choice B) can be performed for multiple reasons. It is often used to detect fetal anomalies and to evaluate for appropriate development. However, this study is not necessary at this time. Signs of fetal distress in the setting of preeclampsia warrant admission to the hospital for a possible induction of labor or cesarean delivery.

A CT of the head (choice C) is not warranted in this patient. Complications of preeclampsia include cerebral infarct and hemorrhage. This patient has no neurologic findings to suggest these complications and therefore, a CT is not necessary.

Serum liver function tests and a complete blood count (choice D) are not critical in addressing the urgent issue of fetal distress. A complication of preeclampsia is HELLP syndrome. This syndrome manifests as hemolysis, elevated liver enzymes, and low platelet count. If this syndrome is suspected, the serum liver function tests and complete blood count are necessary to make the diagnosis.

A urinalysis to evaluate for protein (choice E) is not the most appropriate next step in the management of a pregnancy with signs of fetal distress. Proteinuria is a sign of preeclampsia, and a urinalysis is often ordered in pregnant women with high blood pressure. This type of work up should be pursued in patients who are less critically ill and who do not have pregnancies complicated by fetal distress.

 

 

A 26-year-old financial analyst comes to the office for her 28-week prenatal visit. You have followed her since her 12-week visit and her pregnancy course has been complicated only by some nausea and vomiting late in the first trimester. She is up to date on all of her laboratory work, and an 18-week anatomy scan was within normal limits. She has had appropriate weight gain and continues with prenatal vitamins and iron. She has no medical problems, and does not smoke cigarettes, drink alcohol, or participate in any drug use. At today’s visit she is complaining of some increased fatigue and dependent edema. Her blood pressure is 90/50 mm Hg, fundal height is appropriate and a Doppler of the fetal heart reveals a fetal heart rate of 140 beats per minute. At today’s visit, the most important thing for you to do is

  A. check a blood type
  B. check a thyroid function panel
  C. do a vaginal exam for baseline cervical exam
  D. order a follow-up growth scan
  E. order weekly non-stress tests
Explanation:

The correct answer is A. This patient is at her 28-week visit. At this visit it is imperative to check 2 things: a glucose tolerance test for gestational diabetes if the patient has not already had one, and the patient’s blood type to target Rh-negative patients. At the 28-week visit, all Rh-negative patients need to receive rhoGAM in order to prevent alloimmunization and future hydrops.

There is no need to check a thyroid function panel (choice B). In those patients with underlying thyroid disease (hyper or hypothyroidism), a panel should be checked every trimester. However, this patient has no history of thyroid abnormality.

There is no need to do a vaginal exam (choice C) at this time. The patient has no history of cervical incompetence and gives no history of contractions or preterm labor. The cervix is checked at a the initial prenatal visit and then again if there are any indications of labor.

There is no need to have follow-up growth ultrasounds (choice D) on routine prenatal patients. Twin gestations should have growth scans to assure concordant growth. In addition, some physicians advocate growth scans for diabetic patients at 36 weeks in order to target macrosomia and possible shoulder dystocia.

There is no need to order non-stress tests (choice E) in routine prenatal patients until they reach post-dates (after 40 weeks gestation).

 

 

A 1-day-old male neonate, born by caesarean section, at 35-weeks gestation, is in the neonatal intensive care unit. He has a temperature of 38.8 C (101.8 F) at the time of birth. Physical examination reveals a small but alert boy. The lungs are clear to auscultation. The abdomen is normal without distension. A urinalysis reveals:

A renal ultrasound is performed because of concern for urinary obstruction. The ultrasound reveals normal kidneys, but there are bilateral adrenal masses consistent with hemorrhage. Management of this patient should consist of

  A. ampicillin plus gentamicin
  B. ampicillin plus gentamicin and percutaneous nephrostomies
  C. ampicillin plus gentamicin and a repeat ultrasound in 1 week
  D. gentamicin and percutaneous nephrostomies
  E. a voiding cystourethrogram immediately
Explanation:

The correct answer is C. This neonate has both a urinary tract infection and adrenal hemorrhages. A urinary tract infection in the neonate is treated with intravenous ampicillin and gentamicin. An ultrasound is performed to rule out gross anomalies or obstruction. In this case, an incidental note is made of adrenal hemorrhage which is seen almost exclusively in newborns and is an indicator of birth trauma, stress, anoxia, or dehydration. To ensure that these adrenal masses really are hematomas, a follow-up ultrasound must be performed in 1-2 weeks. A voiding cystourethrogram (VCUG) will be necessary later to evaluate this boy for reflux.

Ampicillin plus gentamicin (choice A) is the antibiotic regimen of choice to treat a urinary tract infection in the neonate. However, the adrenal hemorrhage must also be followed up to exclude adrenal hypertrophy or other adrenal masses.

Ampicillin plus gentamicin is the antibiotic regimen of choice to treat a urinary tract infection in the neonate. The adrenal hemorrhage must also be followed up to exclude adrenal hypertrophy or other adrenal masses. Percutaneous nephrostomies (choice B) are not indicated, as there is no anatomic obstruction.

Gentamicin (choice D) alone is insufficient antibiotic coverage for a urinary tract infection in the neonate. Moreover, percutaneous nephrostomies are not indicated, as there is no anatomic obstruction.

A voiding cystourethrogram (choice E) is not appropriate at this time. Immediate treatment with antibiotics is necessary. Secondarily, the putative adrenal hemorrhages must also be reevaluated with a follow-up ultrasound.

 

 

You are asked to see a 3-week-old infant in the emergency department with a 1-day history of fever. The parents measured his temperature because he “felt warm” to them and found a temperature of 38.3 C (101.0 F). He has been feeding normally, taking 2 ounces of formula every 3-4 hours. He had 6 wet diapers the previous day. Examination shows an active infant with a temperature of 38.8 C (101.8 F). His skin perfusion is good and his physical examination, including examination of his tympanic membranes, is normal. There are no ill household contacts. The most appropriate next step is to

  A. discharge the patient with close outpatient follow up
  B. inquire about the mother’s group B streptococcal status at delivery
  C. obtain the infant’s vaccination history
  D. order a urinalysis and, if negative, do blood and CSF cultures
  E. send blood, urine, and CSF cultures and begin empiric intravenous antibiotic therapy
Explanation:

The correct answer is E. There is absolutely no way to reliably distinguish a self-limited viral illness from sepsis or meningitis in an infant less than 4-weeks of age. Accordingly, all infants with a fever greater than 38 C (100.4 F) in this age group require full evaluation, including admission and parenteral antibiotics. The incidence of sepsis is somewhere between 5-10% with this degree of fever and can be catastrophic if missed, resulting in death or permanent neurologic disability.

Discharging the patient (choice A) is inappropriate because well-appearing infants in this age group may still have a potentially lethal bacterial disease.

At 3 weeks, this infant is at the peak incidence of late onset group B streptococcal disease, but the mother’s group B streptococcal status (choice B) does not correlate well with occurrence of late onset group B streptococcal disease and so it is not relevant here. Maternal group B streptococcal screening is useful in the management of early-onset disease (disease within the first few days of life).

At 3 weeks this infant is unlikely to have received any vaccinations (choice C), with the possible exception of hepatitis B vaccine. He would not in any case have received Haemophilus influenzae or conjugated Streptococcus pneumoniae vaccine, which are the only two that could potentially influence his susceptibility to sepsis or meningitis.

Urinalysis (choice D) is not reliable in excluding urinary tract infection in very young infants, and therefore is not a good screening test to decide if further evaluation is necessary.

 

 

You are notified that one of your patients, a 35-year-old pregnant woman, has gone into labor at 39 weeks. By the time you arrive at the hospital, the baby has already been born during an uneventful delivery, about 2 minutes earlier. You go to examine the baby on the warmer and discover that he has respiratory distress and is becoming cyanotic. You attempt to oxygenate him with a mask, but this does not seem to provide him with much relief. Physical examination shows subcostal and intercostal retractions, absent air entry on the left side, poor air entry on the right side, and “gurgle-like” sounds in the left chest. The heart sounds are best heard in the right hemithorax; the abdomen is flat without organomegaly. The most likely cause of these findings is

  A. aspiration of meconium at delivery
  B. a defect in the left hemidiaphragm
  C. a deficiency of surfactant
  D. dextrocardia and situs inversus
  E. esophageal atresia with distal tracheoesophageal fistula
  F. transposition of the aorta and pulmonary artery
Explanation:

The correct answer is B. This patient most likely has a congenital diaphragmatic hernia, which is when the abdominal contents herniate into the left hemithorax through a congenital defect in the left hemidiaphragm. This causes displacement of the heart into the right hemithorax and pulmonary hypoplasia. If gas has entered the bowel tract, bowel sounds (gurgle-like noises) can be heard in the chest. The displacement of the abdominal contents renders the abdomen flat. In cases of severe pulmonary hypoplasia, administration of oxygen results in only a poor improvement in oxygenation.

Meconium aspiration (choice A) should be accompanied by a history of fetal stress, meconium stained amniotic fluid, or meconium staining of the skin, placenta, or umbilical cord. Physical findings are symmetric, with no displacement of the heart, the breath sounds should be similar on both sides, and there should not be bowel sounds in the chest.

Hyaline membrane disease is caused by a deficiency of surfactant (choice C) and is a rare condition in an infant born at 39 weeks. It typically occurs in infants born before 32 weeks, when surfactant is produced. Surfactant is a phospholipid that prevents alveolar collapse by increasing lung compliance and decreasing surface tension.

It is very unlikely that this patient has dextrocardia and situs inversus (choice D), which are rare positional anomalies where the apex of the heart is on the right (instead of the left) and the abdominal viscera are reversed (the liver is on the left and the stomach is on the right). In cases of dextrocardia and situs inversus, the breath sounds should be basically normal on both sides, and you should not be able to hear bowel sounds in the chest.

Esophageal atresia with distal tracheoesophageal fistula (choice E) usually presents with a history of polyhydramnios, cyanosis with feeding, and increased oropharyngeal secretions. It is unusual for this condition to present so soon after birth with severe respiratory distress.

Transposition of the aorta and pulmonary artery (choice F) is a rare form of congenital heart disease where the aorta arises from the right ventricle and the pulmonary artery arises from the left ventricle. Infants with this disorder are cyanotic from birth and congestive heart failure usually follows. However the breath sounds would be similar on both sides and there would not be bowel sounds in the chest.